You are on page 1of 55

1. Arrange the following components of the B.

The first image of a blackhole has been


universe in an increasing order of their share in captured by Event Horizon Telescope
universe? Project.
1. Stars and comets C. Gravitational Waves are created when
2. Dark energy two black holes orbit each other and
merge.
3. Dark matter
D. A blackhole is a region where gravity
Select the correct option from below:
is weak and therefore light can escape
A. 2>3>1 from blackholes.
B. 1>2>3 Answer: D
C. 3>2>1 Explanation:
D. 2>1>3 Option 1, 2 and 3 are correct.
Answer: A Option 4 is incorrect. A blackhole is a region
Explanation: where gravity is strong and therefore light
cannot escape from blackholes.
Dark energy constitutes 68% of the universe,
dark matter constitutes 27% of the universe A black hole is a place in space where gravity
and stars, comets, planets, blackholes pulls so much that even light cannot get out.
constitute 5% of the universe. Also, the gravity is so strong because matter
has been squeezed into a tiny space.

2. Sunspots are found in which part of the sun?


4. Which of the following pairings are incorrect
A. Chromosphere
regarding countries and their missions to space?
B. Radiative zone A. Apollo–Soyuz Test Project : USA and
C. Photosphere Russia
D. Corona B. Gaganyaan : India
Answer: C C. AstroSat : Nepal
Explanation: D. Both (a) and (c)
Sunspots are found on photosphere. Answer: C
Explanation:
3. Which of the following statements is incorrect • Option C is incorrect. AstroSat
regarding Blackholes? belongs to India, not Nepal. It is the
A. Albert Einstein first predicted the first dedicated Indian astronomy
existence of black holes in 1916, with mission aimed at studying celestial
his general theory of relativity. sources in X-ray, optical and UV
spectral bands simultaneously. • Option d is incorrect. The Wigner
effect also known as the
• Option B is correct. Gaganyaan is an
discomposition effect or Wigner's
Indian crewed orbital spacecraft
disease, is the displacement of atoms in
intended to be the formative spacecraft
a solid caused by neutron radiation.
of the Indian Human Spaceflight
Programme
• Option A is correct. Apollo–Soyuz 6. Pluto is not referred to as a planet. Which of the
was the first crewed international following criteria of planet, Pluto does not
space mission, carried out jointly by satisfy?
the United States and the Soviet Union A. It orbits around a fully-fledged star
in July 1975.
B. It has cleared the neighborhood objects
around its orbit.
5. The maximum mass theoretically possible for C. It has enough gravity to pull itself into
a stable white dwarf star is defined by a spherical shape.
A. Chandrasekhar limit D. Both (a) and (b)
B. Kondo effect Answer: B
C. Butterfly effect International Astronomical Union (IAU), a
D. Wigner effect group of experts, is the authorized body to
define the criteria for any object to be
Answer: A
designated as a planet.
• Option a is correct. Chandrasekhar
In 2006, IAU defines three criteria to classify
limit is the maximum mass
any object as a planet.
theoretically possible for a stable white
dwarf star. A limit which mandates It needs to be in orbit around any fully-
that no white dwarf (a collapsed, fledged star.
degenerate star) can be more massive It needs to have enough gravity to pull itself
than about 1.4 times the mass of the into a spherical shape.
Sun.
It has cleared the neighbourhood around its
• Option b is incorrect. Kondo effect orbit.
describes the scattering of conduction
Pluto has not satisfied third criteria (It has
electrons in a metal due to magnetic
cleared the neighbourhood around its orbit)
impurities.
Therefore, Pluto is not referred to as a planet
• Option c is incorrect. The butterfly anymore.
effect is the sensitive dependence on
initial conditions in which a small
change in one state of a deterministic 7. Recently, Kessler Syndrome was in news. It
nonlinear system can result in large refers to
differences in a later state. A. It is a condition in which a person has
an extra chromosome. 3. Dead satellites
B. It is a developmental disorder that Select the correct option from below:
affects many parts of the body. A. 1 only
C. It is a theoretical scenario in which B. 3 only
Earth's orbit is overpopulated with
objects and debris, preventing the use C. 2 only
of satellites in certain sections of D. 2 and 3
Earth's orbit.
Answer: D
D. It is a complex genetic condition that Explanation:
affects many parts of the body. In
infancy, this condition is characterized Space junk is any piece of machinery or debris
by weak muscle tone. left by humans in space—principally in Earth
orbit. These include dead satellites, fragments
Answer: C of rocket bodies and spacecraft etc. Asteroids
Explanation: don't form part of space debris.
Option c is correct. The Kessler effect,
collisional cascading or ablation cascade, is a
scenario in which the density of objects in 9. Consider the following statements
Low Earth Orbit (LEO) is high enough that regarding Outer Space Treaty
collisions between objects could cause a 1. The United Nations Committee on the
cascade where each collision generates space Peaceful Uses of Outer Space overseas
debris that increases the likelihood of further this treaty.
collisions.
2. No country can place weapons of mass
Option a is incorrect. Turner syndrome is a destruction into the orbit surrounding
condition in which a person has an extra earth.
chromosome.
3. India is a party to this treaty.
Option b is incorrect. Williams syndrome is a Which of the above statements is/ are
developmental disorder that affects many correct?
parts of the body.
A. 1, 2 and 3
Option d is incorrect. Prader-Willi syndrome
is a complex genetic condition that affects B. 1 and 2
many parts of the body. In infancy, this C. 2 and 3
condition is characterized by weak muscle
tone. D. 1 and 3
Answer: A

8. Which of the following can be included as part Explanation: The Outer Space Treaty, formally the
of space debris? Treaty on Principles Governing the Activities of
States in the Exploration and Use of Outer Space,
1. Asteroids
including the Moon and Other Celestial Bodies, is
2. Fragments of rocket bodies a multilateral treaty that forms the basis of
international space law. India has been a party to
this treaty since 1967. According to the treaty, 11. Consider the following statements
No country can place the weapons of mass regarding GSLV MK III
destruction in orbit of earth, moon etc. 1. It is a three-stage heavy lift launch
Makes countries liable for any damages. vehicle.

Forbids any country from claiming a resource from 2. It has has two solid strap-ons, a core
any celestial body. liquid booster and a cryogenic upper
stage.
Not prohibited launching of ballistic missiles
which could be armed with WMD. Which of the above statements is/ are
correct?
A. 1 only
10. Which of the following are features of a
scramjet engine? B. 2 only

1. It does not need to carry oxygen. C. Both 1 and 2

2. It operates at hypersonic speed. D. Neither 1 and 2

3. It will help in accessing the outer space Answer: C


at a cheaper price. Explanation:
Select the correct option from below: Both statements are correct. GSLV MkIII is a
A. 2 and 3 three-stage heavy lift launch vehicle
developed by ISRO. The vehicle has two solid
B. 1 and 2
strap-ons, a core liquid booster and a
C. 1 and 3 cryogenic upper stage. GSLV Mk III is
D. 1, 2 and 3 designed to carry 4 ton class of satellites into
Geosynchronous Transfer Orbit (GTO) or
Answer: D
about 10 tons to Low Earth Orbit (LEO),
Explanation: All the statements are correct. which is about twice the capability of the
Scramjet engine designed by ISRO uses GSLV Mk II.
Hydrogen as fuel and the Oxygen from the
atmospheric air as the Oxidizer. So, Scramjet
12. Consider the following pairs:
engine does not have to carry oxygen.
1. Meteor : a celestial object that enters
Scramjet operates at hypersonic speed.
the atmosphere of the Earth and burns
Scramjet would help to reduce the weight of out completely before reaching the
launch vehicle and increase the speed. It surface.
would help India to do more with a limited
2. Meteorites : able to hit the surface of
space budget.
earth as they are large is size.
Scramjet Engine which would help to
3. Meteoroids : lumps of rock or iron that
minimize the cost.
orbit the sun.
Which of pairs given above is/are correctly How many pairs given above is/are correctly
matched? matched?
A. Only one pair A. Only one pair
B. Only two pairs B. Only two pairs
C. Only three pairs C. Only three pairs
D. None of the pairs D. None of the pairs
Answer: C Answer: C
Explanation: Explanation: All the pairings are correct.
All the statements are correct. Meteoroids are IRS-1B, Indian Remote Sensing satellite-1B,
objects in space that range in size from dust the second of the series of indigenous state-of-
grains to small asteroids. art remote sensing satellites.
When meteoroids enter Earth’s atmosphere
INSAT-3B is an Indian communications
(or that of another planet, like Mars) at high
satellite which was built by the Indian Space
speed and burn up, the fireballs or “shooting
Research Organisation and operated by
stars” are called meteors.
Indian National Satellite System.
When a meteoroid survives a trip through the
atmosphere and hits the ground, it’s called a ResourceSat-2 mission is to support remote
meteorite. sensing data for integrated land and water
resources management at micro level, with
improved spectral and spatial coverage.

14. Which of the following pairings is/ are


correct regarding orbits?
1. Polar Orbit : It is type of low Earth
orbit, as they are at low altitudes
between 200 to 1000 km.
2. Low Earth Orbit : It is an orbit that is
relatively close to Earth's surface.
3. Geo Stationary Orbit : It is a circular
13. Which of the following pairings is/ are geosynchronous orbit 35,786 km in
correct regarding Indian satellites? altitude above Earth's Equator and
following the direction of Earth's
1. IRS-1B : Earth observation satellite
rotation.
2. INSAT-3B : Communication satellite
4. Semi- Synchronous Orbit : It is a near-
3. RESOURCESAT-2 : Remote sensing circular orbit, 26,560 kilometres from
satellite the centre of the Earth.
How many pairs given above is/are correctly Option b is correct. The Kuiper Belt is a ring
matched? of icy bodies just outside of Neptune’s orbit.
A. Only two pairs Pluto is the most famous Kuiper Belt Object.
Option a is incorrect. Oort Cloud is believed
B. Only three pairs
to be a giant spherical shell surrounding the
C. Only one pair rest of the solar system. It is like a big, thick-
D. All the four pairs walled bubble made of icy pieces of space
debris the sizes of mountains and sometimes
Answer: D larger. The Oort Cloud might contain billions,
Explanation: or even trillions, of objects.
All the pairings are correctly matched. Polar Option c is incorrect. Lagrange Points are
orbits are a type of low Earth orbit, as they are positions in space where objects sent there
at low altitudes between 200 to 1000 km. Sun- tend to stay put. At Lagrange points, the
synchronous orbit (SSO) is a particular kind gravitational pull of two large masses
of polar orbit. Satellites in SSO, travelling over precisely equals the centripetal force required
the polar regions, are synchronous with the for a small object to move with them. There
Sun. Low-Earth orbit (often known as LEO) are five special points where a small mass can
encompasses Earth-centered orbits with an orbit in a constant pattern with two larger
altitude of 2,000 km (1,200 mi) or less. Geo masses.
Stationary Orbit is a
Option d is incorrect. Asteroid belt is a region
circular geosynchronous orbit 35,786 km in
between Mars and Jupiter became the
altitude above Earth's Equator and following
asteroid belt.
the direction of Earth's rotation.

16. Consider the following statements with


15. Region of the Solar System that exists beyond reference to Pinaka Extended Range Rocket
the eight major planets. The inner edge begins at System:
the orbit of Neptune and the outer edge continues
outward to nearly 1,000 AU. It contains many 1. It is a Multi-Barrel Rocket-Launcher
small icy bodies, comets and dwarf planets, (MBRL) system
thereby, all remnants from the Solar System ‘s 2. It can achieve a range of up to 37 km
formation.
Which of the above is/are correct?
Above passage to which of the following region:
A. 1 only
A. Oort cloud
B. 2 only
B. Kuiper belt
C. Both 1 and 2
C. Langrange point
D. Neither 1 nor 2
D. Asteroid belt
Answer: A
Answer: B
Explanation:
Explanation:
Statement 1 is correct: It is a Multi-Barrel Rocket- • Trishul (the naval version of Prithvi)
Launcher (MBRL) system. • Nag (anti-tank)
Statement 2 is incorrect: It can achieve a range of
up to 45 km
18. What is India’s Recent Indigenous Move in
Additional Information: the Defence Sector?
• The Pinaka, a Multi-Barrel Rocket- 1. Defence India Startup Challenge
Launcher (MBRL) system named after
2. INS Vikrant: Aircraft Carrier
Shiva’s bow, can fire a salvo of 12 rockets
over a period of 44 seconds. 3. Dhanush: Long-range artillery gun
• The new version is equipped with 4. Arihant: Light Combat Helicopter
advanced technology to enhance its 5. Prachand: Nuclear Submarine
strength. The metal weight is lesser
Select the correct code:
compared to the earlier version.
A. 1, 2 and 3 only
• The newly tested system can achieve a
range of up to 45km which is a big feat for B. 4 and 5 only
the Indian Army. The existing Pinaka C. 2, 4 and 5 only
system, which is already in the Army, has a
range of up to 35-37km. D. 1, 2 and 3 only
Answer: D

17. Consider the following pairs of missile Explanation:


systems: • Defence India Startup Challenge
1. Prithvi - surface-to-air • INS Vikrant: Aircraft Carrier
2. Akash - surface-to-surface • Dhanush: Long-range artillery gun
3. Trishul - the naval version of Prithvi • Arihant: Nuclear Submarine
4. Nag - anti-tank • Prachand: Light Combat Helicopter
Which of the above is/are correctly matched?
A. 1 and 2 only 19. Consider the following statements with
reference to Chief of Defence Staff (CDS)
B. 3 and 4 only
1. The post of CDS was created on the
C. 1 and 3 only
recommendations of Shekatkar committee
D. 2 and 4 only
2. The CDS heads the Department of Military
Answer: B Affairs (DMA) in the Ministry of Defence.
Explanation: Which of the above is/are correct?
• Prithvi (surface-to-surface) A. 1 only
• Akash (surface-to-air) B. 2 only
C. Both 1 and 2 D. 1,2,3,4 and 5
D. Neither 1 nor 2 Answer: D
Answer: C Explanation:
Explanation: • Ballistic missiles of India: Agni, K-4
(SLBM), Prahaar, Dhanush, Prithvi and
Statement 1 is correct: Finally, the post of CDS was
Trishul.
created in 2019 on the recommendations of a
committee of defense experts headed by Lt General • Ballistic missiles, meanwhile, are launched
DB Shekatkar. directly into the upper layers of the earth’s
atmosphere.
Statement 2 is correct: CDS heads the newly
created Department of Military Affairs (DMA) in • They travel outside the atmosphere, where
the Ministry of Defence. the warhead detaches from the missile and
falls towards a predetermined target.
Additional Information:
• They are rocket-propelled self-guided
• The Chief of Defence Staff of the Indian
weapons systems which can carry
Armed Forces is the head and the highest-
conventional or nuclear munitions. They
ranking officer of the Indian Armed
can be launched from aircraft, ships and
Forces.
submarines, and land.
• The President of India is the supreme
commander of the Armed Forces.
21. Consider the following statements with
• The Chief of Defence is the military’s chief
reference to Defence Acquisition Procedure:
executive with operational and strategic
authority over the defence forces and not 1. ‘Make-I’ refers to government-funded
its commander. projects
2. ‘Make-II’ covers industry-funded
programmes.
20. Which of the following is/are the Ballistic
missiles of India? 3. ‘Make-III’ that covers military hardware
that may not be designed and developed
1. Agni
indigenously
2. K-4 (SLBM)
Which of the above is/ are correct?
3. Prahaar
A. 1 only
4. Dhanush
B. 2 and 3 only
5. Trishul
C. 1 and 3 only
Select the correct code:
D. 1, 2 and 3
A. 1 and 2 only
Answer: D
B. 3 and 5 only
Explanation:
C. 4 only
What is the ‘Make’ Category?
• The ‘Make’ category of capital acquisition • Along with buying equipment from Indian
is the cornerstone of the Make in India or international players, DAP 2020 had also
initiative that seeks to build indigenous introduced a chapter on leasing of
capabilities through the involvement of equipment for a limited time.
both public and private sector. • Hiring for shorter-durations, under the
Statement 1 is correct: ‘Make-I’ refers to new powers, will allow the forces to plug
government-funded projects while ‘Make-II’ immediate gaps.
covers industry-funded programmes. Make-I
involved in development of big-ticket platforms
such as light tanks and communication equipment 22. Which of the following is not an Air to Air
with Indian security protocols. missile?
A. Astra
Statement 2 is correct: Make-II category involves
prototype development of military hardware or its B. MICA
upgrade for import substitution for which no C. Sagarika
government funding is provided.
D. Derby
Statement 3 is correct: Another sub-category
under ‘Make’ is ‘Make-III’ that covers military Answer: C
hardware that may not be designed and developed Explanation:
indigenously, but can be manufactured in the Sagarika is an Arihant class submarine.
country for import substitution, and Indian firms
may manufacture these in collaboration with
foreign partners. 23. Consider the following statements with
reference to Air Independent Propulsion (AIP):
Additional Information:
1. Air Independent Propulsion (AIP) is a
Defence Acquisition Procedure (DAP) 2020
mechanism that allows the batteries to be
• The first Defence Procurement Procedure charged even while the boat is submerged.
(DPP) was promulgated in the year 2002.
2. With AIP, the submarine does not have to
• It has since been revised periodically to surface every once in a while.
provide impetus to:
Which of the above is/are correct?
o The growing domestic industry
A. 1 only
and
B. 2 only
o Achieve enhanced self reliance in
defence manufacturing. C. Both 1 and 2
• The armed forces acquire capital assets D. Neither 1 nor 2
according to the Defence Acquisition Answer: A
Procedure (DAP) 2020, which was
Explanation:
introduced in September 2020.
Statement 1 is correct: Air Independent
Propulsion (AIP) is a mechanism that allows the
batteries to be charged even while the boat is 25. Consider the following statements with
submerged. reference to Sindhu Netra Satellite
Statement 2 is incorrect: Even with AIP, the 1. It has been developed by Indian Space
submarine needs to surface every three weeks or Research Organisation (ISRO)
so. 2. The satellite is capable of automatically
identifying the warships and merchant
ships in the Indian Ocean Region (IOR).
24. Consider the following statements with
reference to Project 75I: 3. The satellite enhances India’s surveillance
capabilities on land borders with China
1. Under this project, the Indian Navy intends
and Pakistan.
to acquire six diesel-electric submarines
Which of the above is/are correct?
2. Project 75 (I) is part of the Indian Navy’s 30
year Plan for indigenous submarine A. 1 only
construction. B. 2 and 3 only
3. It will be the developed under the strategic C. 3 only
partnership model
D. 1 and 2 only
Which of the above is/are correct?
Answer: B
A. 1 and 2 only
Explanation:
B. 2 and 3 only
Statement 1 is incorrect: Developed by: Students
C. 1 and 3 only of Bengaluru based PES University under a
D. 1, 2 and 3 contract by Research Centre Imarat, which is a part
of the DRDO.
Answer: D
Statement 2 is correct: The satellite is capable of
Explanation:
automatically identifying the warships and
Statement 1 is correct: Under this project, the merchant ships in the Indian Ocean Region (IOR).
Indian Navy intends to acquire six diesel-electric
Statement 3 is correct: The satellite enhances
submarines, which will also feature advanced Air-
India’s surveillance capabilities on land borders
independent propulsion (AIP) systems
with China and Pakistan.
Statement 2 is correct: Project 75 (I), approved in
2007, is part of the Indian Navy’s 30 year Plan for
indigenous submarine construction. 26. Consider the following statement with
reference Nirbhay Missile:
Statement 3 is correct: It will be developed under
the strategic partnership model which was 1. Nirbhay is a long-range subsonic cruise
promulgated in 2017 to boost indigenous defence missile
manufacturing. 2. The Aeronautical Development
Establishment (ADE), had designed and
developed Nirbhay
3. The missile is capable of carrying both
conventional and nuclear warheads 27. Consider the following statement with
Which of the above is/are correct? reference INS Arihant:
A. 1 and 2 only 1. It is India’s first indigenous nuclear
powered ballistic missile
B. 3 only
2. It is a submarine built under the secretive
C. 1 only
Advanced Technology Vessel (ATV)
D. 1, 2 and 3 project
Answer: D Which of the above is/are correct?
Explanation: A. 1 only
Statement 1 is correct: Nirbhay is a long-range B. 2 only
subsonic cruise missile.
C. Both 1 and 2
Statement 2 is correct: The Aeronautical
D. Neither 1 nor2
Development Establishment (ADE), which is part
of the Defence Research and Development Answer: C
Organisation, designed and developed Nirbhay. Explanation:
Statement 3 is correct: The missile is capable of Statement 1 and 2 are correct: It is India’s first
carrying both conventional and nuclear warheads. indigenous nuclear powered ballistic missile
Additional Information: capable submarine built under the secretive
Advanced Technology Vessel (ATV) project
• Nirbhay is a long range, all-weather,
subsonic cruise missile designed and Additional Information:
developed in India by the Defence About INS Arihant
Research and Development Organisation
• It was launched in 2009 and commissioned
(DRDO).
in 2016.
• The missile can be launched from multiple
• It is India’s first indigenous nuclear
platforms and is capable of carrying
powered ballistic missile capable
conventional and nuclear warheads.
submarine built under the secretive
• It is a two-stage missile powered by a Solid Advanced Technology Vessel (ATV)
rocket motor booster. project, which was initiated in the 1990s.
• It is capable of carrying warheads of up to • INS Arihant and its class of submarines are
300kg at a speed of 0.6 to 0.7 Mach classified as ‘SSBN’, which is the hull
(subsonic). classification symbol for nuclear powered
• It has an operational range of 1000 km (long ballistic missile carrying submarines.
range). o While the Navy operates the vessel,
• The Nirbhay cruise missile is an Indian the operations of the SLBMs from
version of the American Tomahawk. the SSBN are under the purview of
India’s Strategic Forces Command,
which is part of India’s Nuclear • One of the most low-carbon energy sources
Command Authority. • It also has one of the smallest carbon
• INS Arihant is presently armed with K-15 footprints
SLBM with a range of 750 km. • It's one of the answers to the energy gap
o INS Arihant can carry a dozen K-15 • It's essential to our response to climate
missiles on board. change and greenhouse gas emissions
• Reliable and cost-effective
28. What are the advantages of electricity Disadvantages of using nuclear energy:
produced using nuclear energy?
• It is not a clean source of energy because
1. It is a source of clean energy.
very harmful nuclear radiations are
2. It helps in the development of a country’s produced in the process.
economy without adversely contributing to
• The waste causes environmental pollution.
climate change.
3. It does not emit any greenhouse gases.
29. Which of the following is/ are uses of nuclear
4. It can be built in urban or rural areas energy?
Select the correct code: 1. Nuclear power is used for space
A. 1 and 2 only explorations.
B. 3 and 4 only 2. It is used for providing potable water
through desalination
C. 4 only
3. Nuclear radiation is used in the treatment
D. 1, 2, 3 and 4
of food by killing bacteria, insects, and
Answer: - D parasites that cause illness.
Explanation: Select the correct code:
• Statement 1 is correct: It is a source of A. 1 only
clean energy.
B. 1 and 3 only
• Statement 2 is correct: It helps in the
development of a country’s economy C. 2 only
without adversely contributing to D. 1, 2 and 3
climate change.
Answer: D
• Statement 3 is correct: It does not emit Explanation:
any greenhouse gases.
Uses of nuclear energy are:
• Statement 4 is correct: It can be built in
• Nuclear power is used for space
urban or rural areas
exploration.
Additional Information:
• It can be built in urban or rural areas.
The advantages of nuclear power are:
• Used for providing potable water through • Una District – Himachal Pradesh
desalination. • Gondia District – Maharashtra
• Used in cancer treatment, Nuclear
radiation is used in the treatment of food by
killing bacteria, insects, and parasites that 31. Which of the following is used as moderator
cause illness. in the nuclear reactor?
A. Plutonium
• Used for sterilizing medical equipment.
B. Thorium
• Nuclear energy could play a major role in
transportation by acting as a substitute for C. Graphite
fossil fuels. D. Berilium
Answer: C
30. Which of the following is not a location for Explanation:
Uranium resources?
• Moderators are used for reducing the speed
A. Nalgonda District – Telangana of fast neutrons released from the fission
B. East Singhbhum District – Jharkhand reaction and making them capable of
sustaining a nuclear chain reaction.
C. Chhaya – Mithi – Gujarat
• Usually, water, solid graphite, and heavy
D. West Khasi Hills District – Meghalaya
water are used as a moderator in nuclear
Answer: C reactors.
Explanation:
The Nuclear Power Corporation of India Limited 32. Consider the following statements:
(NPCIL) has abandoned its plan to set up a 6000
MW nuclear power plant at Chhaya-Mithivirdi 1. Atomic explosion is carried out by via
village of Bhavanagar district in Gujarat. nuclear reaction in which there is net
change in mass of the product
Location for uranium resource are:
2. The release in energy in nuclear fission is
• Tummalapalle (Kadapa District) –
consistent with the fact that uranium has
Andhra Pradesh more mass per nucleon than either of the
• Nalgonda District – Telangana two fragments
• East Singhbhum District – Jharkhand Which of the above is/are correct?
• West Khasi Hills District – Meghalaya A. 1 only
• Udaipur District – Rajasthan B. 2 only
• Yadgir District – Karnataka C. Both 1 and 2
• Rajnandgaon (District) – Chhattisgarh D. Neither 1 nor 2
• Sonbhadra district – Uttar Pradesh Answer: B

• Rudraprayag District – Uttarakhand Explanation:


• Statement 1 is correct: Atomic fusion is the fusing of two or more smaller
explosion is carried out via nuclear atoms into a larger one.
reaction in which there is net change in Statement 2 is correct: The energy we get in
mass of the product and reactant and nuclear reaction comes from the mass of the fuel
this change in mass gets converted into
energy due to which a huge amount of Statement 3 is incorrect: When a neutron leaves
energy in the form of heat is released. the nucleus, it’s mass decreases.

• Statement 2 is correct: The release in


energy in nuclear fission is consistent 34. Consider the following statements:
with the fact that uranium has more
1. India has 25% of World’s Uranium reserves
mass per nucleon than either of the two
and only 2% of the World’s Thorium
fragments
reserves.
2. Thorium is not a fissile material
33. Consider the following statements:
3. Thorium cannot be converted into
1. The beneficial aspect of nuclear fission is Uranium – 233
the ability to produce more energy than
Which of the above is/are correct?
nuclear fusion
A. 1 only
2. The energy we get in nuclear reaction
comes from the mass of the fuel B. 2 and 3 only

3. When a neutron leaves the nucleus, its C. 2 only


mass increases. D. 1 and 3 only
Which of the above is/are correct? Answer: C
A. 1 only • Statement 1 is incorrect: India has 2%
B. 2 and 3 only of World’s Uranium reserves and 25%
of the World’s Thorium reserves.
C. 2 only
• Statement 2 is correct: Thorium is not
D. 1 and 3 only
a fissile material
Answer: C
• Statement 3 is incorrect: Thorium can
Explanation: be converted into Uranium – 233
Statement 1 is incorrect: Additional information:
• The beneficial aspect of nuclear fission is • A nuclide that is capable of undergoing
the ability to release tremendous amounts fission after capturing low-energy thermal
of energy, not more than nuclear fusion. (slow) neutrons.
• The main difference between these two • Although sometimes used as a synonym
processes is that fission is the splitting of an for fissionable material, this term has
atom into two or more smaller ones while acquired its more-restrictive interpretation
with the limitation that the nuclide must be 36. Consider the following statements with
fissionable by thermal neutrons. reference to Uranium Enrichment:
• With that interpretation, the three primary 1. Natural uranium is only 0.7% U-235, the
fissile materials are uranium-233, uranium- fissionable isotope.
235, and plutonium-239. 2. Centrifugal separators are used in uranium
• This definition excludes natural uranium enrichment.
and depleted uranium that have not been 3. The enriched uranium fuel used in fission
irradiated, or have only been irradiated in reactors cannot be used to make a bomb.
thermal reactors.
Which of the above is/are correct?
A. 1 and 2 only
35. Which of the following Nuclear Reactors and
B. 2 and 3 only
its location in India is incorrectly matched?
C. 1 and 3 only
State Nuclear Plant
D. 1,2 and 3
Tamil Nadu Kudankulam Answer: D
Kalpakkam
Explanation:
Gujarat Kakrapar • Statement 1 is correct: Natural
uranium is only 0.7% U-235, the
Uttar Pradesh Narora
fissionable isotope.
Maharashtra Kaiga • Statement 2 is correct: Centrifugal
separators are used in uranium
Answer: D
enrichment.
Explanation:
• Statement 3 is correct: The enriched
State Location uranium fuel used in fission reactors
cannot be used to make a bomb.
Rajasthan Rawatbhata Additional Information:

Tamil Nadu Kudankulam Uranium Enrichment

Kalpakkam • Natural uranium is only 0.7% U-235, the


fissionable isotope.
Gujarat Kakrapar
• The other 99.3% is U-238 which is not
Uttar Pradesh Narora fissionable.
• The uranium is usually enriched to 2.5-
Karnataka Kaiga
3.5% U-235 for use in light water reactors.
Maharashtra Tarapur • Centrifugal separators are used in uranium
enrichment.
• The enriched uranium fuel used in fission • Moderators are like accelerators
reactors cannot be used to make a bomb. • Control Rods are like brakes
• It takes enrichment to over 90% to obtain • Absorbing more neutrons in a control rod
the fast chain reaction necessary for means that there are fewer neutrons
weapons applications. available to cause fission.
• Enrichment to 15-30% is typical for breeder • So pushing the control rod deeper into the
reactors. reactor will reduce its power output, and
extracting the control rod will increase it.
37. Consider the following statements:
1. Control Rods slow down neutrons 38. Control rods are composed of which of the
following chemical elements?
2. Moderators absorb neutrons
1. Boron
Which of the above is/are correct?
2. Silver
A. 1 only
3. Indium
B. 2 only
4. Cadmium.
C. Both 1 and 2
Select the correct code:
D. Neither 1 nor 2
A. 1 and 2 only
Answer: D
Explanation: B. 4 only

• Statement 1 is incorrect: Moderators C. 3 and 4 only


slow down neutrons D. 1,2,3 and 4
• Statement 2 is incorrect: Control Rods Answer: D
absorb neutrons Explanation:
Additional Information: • Control rods are composed of chemical
• Control Rods or Reactivity control elements such as boron, silver, indium
and cadmium.
o The power output of the reactor is
adjusted by controlling how many
neutrons are able to create more 39. Consider the following statements with
fissions. reference to Thermal Reactors and Fast Neutron
o Control rods that are made of a Reactors:
neutron poison are used to absorb 1. Thermal Reactors contain neutron
neutrons. moderator materials that slow neutrons.
• Moderators slow down neutrons 2. Fast Neutron Reactors do not have a
• Control Rods absorb neutrons neutron moderator
3. Thermal reactors use slowed or thermal • Irrespective of reactor type, the uranium
neutrons to keep up the fission of their fuel. dioxide (UO2) in the form of rod or pallet is
commonly used as nuclear fuel.
4. Thermal Reactors are more difficult to
build and more expensive to operate
Which of the above is/are correct? 40. Consider the following statements:
A. 1 and 3 only 1. Dry nanotechnology is used to
manufacture structures in coal and silicon
B. 2 and 4 only
2. Wet nanotechnology is based on biological
C. 1, 2 and 3 only
systems present in an aqueous
D. 4 only environment
Answer: C Which of the above is/are correct?
• Statement 1 is correct: Thermal A. 1 only
Reactors contain neutron moderator
B. 2 only
materials that slow neutrons.
C. Both 1 and 2
• Statement 2 is correct: Fast Neutron
Reactors do not have a neutron D. Neither 1 nor 2
moderator Answer: - C
• Statement 3 is correct: Thermal Explanation:
reactors use slowed or thermal
neutrons to keep up the fission of their • Statement 1 is correct: Dry
fuel. nanotechnology :It is used to
manufacture structures in coal, silicon,
• Statement 4 is incorrect: Fast Neutron inorganic materials, metals and
Reactors are more difficult to build and semiconductors that do not work with
more expensive to operate humidity.
Additional Information: • Statement 2 is correct: Wet
Similarities between thermal reactor and fast nanotechnology: It is based on
reactor biological systems present in an
aqueous environment — including
• Both are nuclear fission reactors (these are
genetic material, membranes, enzymes
not nuclear fusion reactors).
and other cellular components.
• Enrichment of natural uranium with U-235
isotopes is required in both types of
reactors; however, the percentage of such 41. Consider the following with reference to
enrichment varies widely. PHWR can be Carbon Nanotubes:
operated at a natural percentage of U-235. 1. It is used in electric wires to reduce losses
• Chain reaction is very much desired to 2. It can replace silicon made transistors as
continue heat generation irrespective of the they are small and emit less heat
type of reactor.
3. It is used in solar cell i.e. in a highly tangled-up and
Which of the above is/ are correct? agglomerated form. For CNTs to
unfold their particular properties they
A. 1 only need to be untangled and spread
B. 2 and 3 only evenly in the substrate.
C. 3 only • Another requirement is that CNTs
need to be chemically bonded with the
D. 1,2 and 3 substrate, e.g. a plastic material.
Answer: D • For that purpose, CNTs are
Explanation: functionalized, i.e. their surface is
chemically adapted for optimal
• Used in electric wires to reduce losses incorporation into different materials
• It can replace silicon made transistors and for the specific application in
as they are small and emit less heat and question.
it can revolutionise electronics • Carbon nanotubes can also be spun
• Can be used in solar cell into fibers, which not only promise
Additional information: interesting possibilities for specialty
textiles but may also help realize a
What are carbon nanotubes?
particularly utopian project – the space
• Carbon nanotubes (CNTs) are cylindrical elevator.
molecules that consist of rolled-up sheets of
single-layer carbon atoms (graphene).
42. Consider the following statements with
• They can be single-walled (SWCNT) with a
reference to Graphene:
diameter of less than 1 nanometer (nm) or
multi-walled (MWCNT), consisting of 1. Graphene is a one-atom-thick sheet of
several concentrically interlinked carbon atoms
nanotubes, with diameters reaching more 2. Graphene is the world’s strongest material
than 100 nm.
3. It cannot be used for making heat-
• Their length can reach several micrometers spreading solutions
or even millimeters.
Which of the above is/are correct?
Applications of carbon nanotubes and their uses
A. 1 only
• CNTs are well-suited for virtually any
B. 2 and 3 only
application requiring high strength,
durability, electrical conductivity, C. 3 only
thermal conductivity and lightweight D. 1 and 2 only
properties compared to conventional
Answer: D
materials.
• Currently, CNTs are mainly used as Explanation:
additives to synthetics. CNTs are • Statement 1 is correct: Graphene is a
commercially available as a powder, one-atom-thick sheet of carbon atoms
• Statement 2 is correct: Graphene is the C. 1 only
world’s strongest material D. 1, 2, 3 and 4
• Statement 3 is incorrect: Graphene is
Answer: D
the most heat conductive found to
date. As graphene is also strong and Explanation:
light, it means that it is a great material Applications of Graphene:
for making heat-spreading solutions,
• anti-corrosion coatings and paints,
such as heat sinks or heat dissipation
films. • efficient and precise sensors,
• faster and efficient electronics,
43. Which of the following is not a feature of • flexible displays,
Graphene?
• efficient solar panels,
A. Higher Capacity
• faster DNA sequencing,
B. Faster Charging
• drug delivery, and more.
C. Low temperature range
D. Flexibility
45. Gold Nanoparticles are used in which of the
Answer: C following?
Explanation: 1. Therapeutic imaging
Feature of graphene are: 2. Cholesterol lowering drug
Select the correct code:
A. 1 only
B. 2 only
C. Both 1 and 2
D. Neither 1 nor 2

44. Which of the following are applications of Answer: C


Graphene? Explanation:
1. Anti-corrosion coatings and paints • Statement 1 is correct: Due to the
2. Efficient and precise sensors optical properties of gold
nanoparticles, they are especially
3. Flexible displays
utilized in ultrasensitive detection and
4. Faster DNA sequencing • imaging-based therapeutic techniques
Select the correct code: required for the treatment of lethal
diseases, such as cancer.
A. 1 and 2 only
• Statement 2 is correct: Functionalized
B. 3 and 4 only gold nanoparticles interact with free
cholesterol to produce H2O2 in C. Nanobarcodes are used to tag individual
proportion to the level of cholesterol products and trace outbreaks.
visually being detected. D. None of the above
Additional information: Answer: A
What are Gold Nanoparticles? Explanation:
• Metallic NPs have been efficiently • Statement 1 is incorrect: Nano-
exploited for biomedical applications and encapsulating can improve the
among them, GNPs are found to be solubility of vitamins, antioxidants,
effective in biomedical research. healthy omega, etc.
• And NPs are those materials that are at • Statement 2 is correct: Nano-fibres
least one dimension smaller than 100 made of lobster shells or organic corn
nanometers. can allow for antimicrobial packaging
while being biodegradable.
• NPs have a high surface-to-volume ratio
and they can provide the tremendous • Statement 3 is correct: Nanobarcodes
are used to tag individual products
driving force for diffusion, especially at
and trace outbreaks.
elevated temperatures.
• GNPs are melted at much lower
temperatures (300 °C) than bulk gold (1064 47. Consider the following statements with
°C). reference to Nanocapsules:

• NPs have been found to impart various 1. The nanocapsule can enable effective
desirable properties to different day-to-day penetration of herbicides into the targeted
products. part of the plant.

• For example, GNPs are found to have 2. This ensures a faster and constant release of
greater solar radiation absorbing ability the necessary substance to the plants with
than the conventional bulk gold, which minimized environmental pollution.
makes them a better candidate for use in Which of the above is/are correct?
the photovoltaic cell manufacturing
A. 1 only
industry.
B. 2 only
C. Both 1 and 2
46. Which of the following statements is
incorrect? D. Neither 1 nor 2

A. Nano-encapsulating can improve the Answer: A


solubility of vitamins but not antioxidants Explanation:
B. Nano-fibres made of lobster shells or • Statement 1 is correct: The nanocapsule
organic corn can allow for antimicrobial can enable effective penetration of
packaging while being biodegradable. herbicides, chemical fertilizers, and
genes into the targeted part of the nanotechnology. The objectives
plant. include the promotion of research,
• Statement 2 is incorrect: This ensures a infrastructure development to support
slow and constant release of the the research, develop nanotechnology,
necessary substance to the plants with human resources, and international
minimised environmental pollution. collaborations.

48. Consider the following statements: 49. Consider the following statements:
1. Nano Science and Technology Initiative 1. Raman Spectroscopy is a non-destructive
(NSTI) focuses on issues related to chemical analysis technique
infrastructure development 2. Raman Scatter is a light scattering
2. Nano Science and Technology Mission technique
(NSTM) is an umbrella programme that 3. At times Raman Spectroscopy was used to
aims to promote research and development detect RNA viruses present in saliva
in nanotechnology. samples
Which of the above is/are correct? Which of the above is/are correct?
A. 1 only A. 1 and 2 only
B. 2 only B. 2 and 3 only
C. Both 1 and 2 C. 1 and 3 only
D. Neither 1 nor 2 D. 1, 2 and 3
Answer: C Answer: D
Explanation: Explanation:
Nano Science and Technology Initiative (NSTI)
• Statement 1 is correct: Raman
• Statement 1 is correct: It was set up by Spectroscopy is a non-destructive
the Department of Science and chemical analysis technique which
Technology (DST) in 2001 to focus on provides detailed information about
issues related to infrastructure chemical structure, phase and
development, research and application • polymorphy, crystallinity and
programmes related to nanomaterials molecular interactions. It is based upon
including drugs, drug delivery, gene the interaction of light with the
targeting and DNA chips. chemical bonds within a material.
Nano Science and Technology Mission (NSTM) • Statement 2 is correct: Raman Scatter is
a light scattering technique, whereby a
• Statement 2 is correct: NSTM,
molecule scatters incident light from a
launched in 2007, is an umbrella
high intensity laser light source. Most
programme that aims to promote
of the scattered light is at the same
research and development in
wavelength (or color) as the laser
source and does not provide useful
information – this is called Rayleigh 51. Consider the following statements:
Scatter. However a small amount of
1. Nanoparticles are used to reduce bacteria
light (typically 0.0000001%) is
on produce
scattered at different wavelengths (or
colors), which depend on the chemical 2. Nanoemulsions based on titanium dioxide
structure of the analyte – this is called – used as antimicrobial agents
Raman Scatter. Which of the above is/are correct?
• Statement 3 is correct: At times Raman
A. 1 only
Spectroscopy was used to detect RNA
viruses present in saliva samples B. 2 only
C. Both 1 and 2
50. Which of the following is not an objective of D. Neither 1 nor 2
the Nano Mission?
Answer: D
A. Establishment of R&D in Nanoscience
Explanation:
Applications
• Statement 1 is incorrect:
B. Establishment of Development Centre for
Nanoemulsions – to reduce bacteria on
Nanosciences
produce. Nanoemulsion based edible
C. Human Development in Nanotechnology nanocoatings containing flavor and
D. None of the above coloring ingredients, antioxidants,
enzymes, antimicrobials, and
Answer: D antibrowning agents can be used to
Explanation: coat foods such as meats, dairy
The Government of India launched the Nano products such as cheese, fresh
Mission in 2007 under the Department of Science produce, and fresh cuts including fruit
and Technology. The Ministry of Science and and vegetables and confectionaries to
Technology allocated up to Rs 1000 crores to this improve their shelf life. The
mission to fulfill its following objectives: nanoemulsion coatings can also
prevent moisture and gas exchange,
• Basic Promotion of Nanotechnology minimize moisture loss and oxidation
• Infrastructure Development of foods
• Statement 2 is incorrect:
• Establishment of R&D in Nanoscience
Nanoparticles based on titanium
Applications
dioxide – used as antimicrobial agents.
• Establishment of Development Centre for Titanium oxide nanoparticles (TiO2
Nanosciences NPs) also proved to have a good
• Human Development in Nanotechnology antimicrobial effect, since its
photocatalytic properties generate free
• International Collaborations
radical oxides and peroxides with
potent antimicrobial activity and What are the 2 unique properties of Stem Cells?
broad reactivity against many • A stem cell is an immature or unspecialized
microbial pathogens. cell that can
• Split to form similar cells
52. Consider the following statements with • Develop into different specialized
reference to Stem Cells:
cells that perform a distinct
1. A stem cell is an immature or unspecialized function.
cell that can split to form similar cells
What are the Different Types of Stem Cells?
2. Stem cells can develop into different • Stem cells are classified into 2 main
specialized cells that perform a distinct categories –
function.
• Classification based on the formation of
3. Induced pluripotent stem cells (iPSC’s) are cells at different phases of human lives
not found in the body and
Which of the statements given above is/are • Classification based on its ability to
correct? form into different specialized cells.
A. 1 and 2 only
B. 2 and 3 only 53. Which of the following is not an advantage of
C. 1 and 3 only CRISPR technology?
D. 1, 2 and 3 A. Inexpensive
Answer: D B. Cancer Treatments
Explanation: C. Pest Resilient Crops
Statement 1 is correct: A stem cell is an immature D. Off Target Effects
or unspecialized cell that can split to form similar Answer: D
cells
Explanation:
Statement 2 is correct: Stem cells can develop into
Advantages of CRISPR:
different specialized cells that perform a distinct
function. • Curing Genetics Disease
Statement 3 is correct: Induced pluripotent stem • Inexpensive
cells (iPSC’s) are not found in the body • Cancer Treatments
Additional information: • Pest Resilient Crops
What are Stem Cells?
• Simple to Amend Target
• Special human cells that have the capability
• Drug Research
to develop into wide-ranging types of cells
in the human body, from muscle cells to Disadvantages of Crispr
brain cells, are called stem cells. • Not Efficient
• Time Consuming o Egg cell is placed into the uterus
after a few divisions in it. The cell is
• Off Target Effects
allowed to develop into a fetus that
• Research and Responsibility is genetically identical to the donor
of the original nucleus.
54. Consider the following statements: • Therapeutic cloning, which creates
1. Gene cloning creates copies of genes or embryonic stem cells. Researchers hope to
segments of DNA. use these cells to grow healthy tissue to
replace injured or diseased tissues in the
2. Reproductive cloning creates embryonic human body.
stem cells.
o In therapeutic cloning, the aim is to
3. Therapeutic cloning creates copies of whole clone cells that make particular
animals organs or types of tissue.
Which of the statements given above is/are o Egg is placed in a Petri dish to
correct?
develop into embryonic stem cells
A. 1 only which have shown potential for
B. 2 and 3 only treating several ailments.

C. 3 only o It is also called somatic cell nuclear


transfer or research cloning.
D. 1 and 2 only
o In this technique, the resultant
Answer: A embryo is allowed to grow for 14
Explanation: days.
Statement 1 is correct: Gene cloning, which creates o Its stem cells would then develop
copies of genes or segments of DNA. into human tissue or a complete
Statement 2 is incorrect: Reproductive cloning, human organ for transplant.
which creates copies of whole animals
Statement 3 is incorrect: Therapeutic cloning, 55. Consider the following statements with
which creates embryonic stem cells. reference to Alpha Fold:
Additional information: 1. Alpha Fold is an Artificial Intelligence-
based protein structure prediction tool.
• Gene cloning, which creates copies of genes
or segments of DNA. 2. It is based on a computer system called
Deep Neural Network.
• Reproductive cloning, which creates copies
of whole animals. 3. Alpha Fold is fed with protein sequences as
input.
o In this we actually reproduce not
organs but entire beings(donors) Which of the statements given above is/are
from where we got genetic correct?
information. A. 1 only
B. 2 and 3 only B. 2 only
C. 1 and 2 only C. Both 1 and 2
D. 1, 2 and 3 D. Neither 1 nor 2
Answer: D Answer: D
Explanation: Explanation:
Statement 1 is correct: AlphaFold is an Artificial Statement 1 is incorrect: In Somatic Gene Therapy
Intelligence-based protein structure prediction effects will not be transferred to next generation
tool. Statement 2 is incorrect: In Germline Gene
Statement 2 is correct: It is based on a computer Therapy effects are transferred to next generation.
system called Deep Neural Network. Additional information:
Statement 3 is correct: AlphaFold is fed with • Gene Therapy: Gene therapy refers to the
protein sequences as input. method wherein a genetic defect is treated
Additional information: by inserting a functional gene or section of
DNA in cells. The functional gene inserted
• DeepMind and EMBL’s European
into an individual or an embryo
Bioinformatics Institute (EMBL-EBI) have
compensates for the non-functional gene
partnered to create AlphaFold DB to make
present.
these predictions freely available to the
scientific community. • The newly inserted DNA corrects the effect
of the mutated gene. Gene therapy was
• In CASP14, AlphaFold was the top-ranked
developed in 1972. The first therapeutic
protein structure prediction method by a
gene therapy was given to a four year old
large margin, producing predictions with
girl in 1990 to treat adenosine deaminase
high accuracy.
(ADA) deficiency.
• While the system still has some limitations,
• Gene Therapy Types: On the basis of types
the CASP results suggest AlphaFold has
of cells involved in the gene transfer, gene
immediate potential to help us understand
therapy is of two types:
the structure of proteins and advance
biological research. o Somatic gene therapy: Here the
desired gene is transferred to a
somatic cell. The gene is not
56. Consider the following statements: transferred to the offspring.
1. In Somatic Gene Therapy effects are
o Germline gene therapy: Here the
transferred to next generation
desired gene is introduced in the
2. In Germline Gene Therapy effects will not germ cell. The gene gets transferred
be transferred to next generation from one generation to another
Which of the above is/are correct? generation.

A. 1 only
57. Which of the following is/are the applications diseases like diabetes, Parkinson’s disease
of Somatic Cell Nuclear Transfer? etc. In 2014, the New York Stem Cell
1. Stem cell research Foundation was successful in creating
SCNT stem cells derived from adult
2. Organ transplants
somatic cells. One of these lines of stem
3. Therapeutic applications cells was derived from the donor cells of a
4. Reproductive Cloning type 1 diabetic. These insulin producing
cells could be used for replacement therapy
5. Preservation of endangered species in diabetics, demonstrating real SCNT stem
Select the correct code: cell therapeutic potential.
A. 1 and 2 only 4. Reproductive cloning- Somatic Cell
Nuclear Transfer technique is currently the
B. 3, 4 and 5 only
basis for cloning animals (such as the
C. 5 only famous Dolly the sheep), and has been
D. 1, 2, 3, 4 and 5 theoretically proposed as a possible way to
clone humans. But there are moral and
Answer: D
ethical objections against reproductive
Explanation: cloning. In 2018, the first successful
Potential Applications of Somatic Cell Nuclear cloning of primates using somatic cell
Transfer: nuclear transfer, (the same method as Dolly
1. Stem cell research- Somatic cell nuclear the sheep), was successfully done with the
transplantation is being used in stem cell birth of two live female clones .
research. These cells are deemed to have a 5. Preservation of endangered species-
pluripotent potential because they have the Interspecies nuclear transfer (iSCNT) is a
ability to give rise to all of the tissues found means of somatic cell nuclear transfer used
in an adult organism. This could be used in to facilitate the rescue of endangered
therapies or disease research. This allows species, or even to restore species after their
stem cells to create any cell type, which extinction.
could then be transplanted to replace
damaged or destroyed cells.
58. Aedes aegypti mosquito is the carrier of which
2. Organ transplants- Another application of of the following diseases?
SCNT stem cell research to generate tissues
1. Zika
or even organs for transplant into the
specific patient. The resulting cells would 2. Dengue
be genetically identical to the somatic cell 3. Chikungunya
donor, thus avoiding any complications
4. Nile
from immune system rejection.
Select the correct code:
3. Therapeutic applications- SCNT have
ample scope of success in therapy and A. 1 only
curing diseases. It can be used to treat
B. 2 and 3 only • Augmented Reality (AR) and Virtual
Reality (VR)
C. 4 only
• Artificial Intelligence
D. 1, 2 and 3 only • 3D reconstruction
Answer: D • Blockchain Technology
• Internet of Everything
Explanation:
Additional info:
• The Aedes aegypti mosquito is the carrier
of diseases such as Zika, dengue and • Metaverse is a term used to describe a
chikungunya. virtual world that is created and run
• Genetically modified mosquitoes involve entirely using digital technology. It allows
producing transgenic male Aedes aegypti users to immerse themselves in a 3D
mosquito, which carries a new gene fatal environment, interact with other users, and
only to female mosquitoes. participate in various activities.

• After a few generations, the female • The concept of Metaverse is becoming


population will be drastically reduced. increasingly popular due to the growing
interest in virtual reality and augmented
• Transgenic males do not bite and the reality technologies.
modified genes are said to be harmless to
humans. • The potential applications of Metaverse are
vast and range from gaming to education,
healthcare, and social media. It is believed
59. Which of the following is/are main Metaverse that the development of Metaverse could
components? lead to a significant shift in the way we
1. Augmented Reality (AR) and Virtual interact with technology and each other.
Reality (VR)
2. Artificial Intelligence 60. Consider the following statements with
reference to Augmented Reality:
3. 3D reconstruction
1. AR replaces actual world environment
4. Blockchain Technology
with 3D digital elements
5. Internet of Everything
2. AR combines both the real-world and
Select the correct code: the virtual.
A. 1 and 2 only Which of the statements given above is/are
B. 3, 4 and 5 only correct?

C. 1 only A. 1 only

D. 1, 2, 3, 4 and 5 B. 2 only

Answer: D C. Both 1 and 2

Explanation: D. Neither 1 nor 2

Main Metaverse components: Answer: B


Explanation: D. 1, 2, 3 and 4
Statement 1 is incorrect: AR does not replace Answer: D
actual world environment with 3D digital Explanation:
elements.
Advantages of Cloud computing
Statement 2 is correct: AR combines both the real-
• Low capital expenditure
world and the virtual. Users of AR are still able to
sense the real world around them. • Flexible resources
Additional info: • Economies of scale
● Augmented Reality (AR) is the technology • Improved disaster recovery and
that superimposes an image onto a user’s reliability
view of the real world and enhances it with
Additional info:
sound, touch, and even smell.
What is Cloud Computing?
● It is a combination of the real scene viewed
by the user and a virtual scene generated by ● Simply put, cloud computing is the
the computer. AR is a technology that is delivery of computing services—including
going to blur the lines of reality. servers, storage, databases, networking,
software, analytics, and intelligence—over
● Augmented Reality has moved beyond the Internet (“the cloud”)
headsets and gaming and permeated into
numerous industries. ● It is the pool of shared resources such as
networks, servers, storage, applications,
● In general terms, Augmented Reality is and services that can be provided to the
increasingly being adopted for a variety of consumer rather than the consumer
uses like assembly, maintenance, repair, managing them on her own which is costly
education, training, retail showcasing, and and time-consuming.
diagnostics.
● Rather than owning their own computing
infrastructure or data centres, companies or
61. Which of the following is/are the advantages individuals can rent access to storage (or
of Cloud Computing? application or services) from a cloud
1. Low capital expenditure service provider.

2. Flexible resources
3. Economies of scale 62. Consider the following statements with
reference to Edge Computing:
uoImproved disaster recovery and reliability
1. The main goal of edge computing is to
Select the correct code:
increase latency requirements
A. 1 only
2. One of the best ways to implement
B. 2 and 4 only edge computing is in smart home
C. 3 only devices.
3. It also has use in the cloud gaming ● Edge computing can bring the data storage
industry. and processing centers close to the smart
Which of the statements given above is/are home and reduce backhaul costs and
correct? latency.

A. 1 only ● Another use case of edge computing is in


the cloud gaming industry. Cloud gaming
B. 2 and 3 only
companies are looking to deploy their
C. 3 only servers as close to the gamers as possible.
D. 1 and 2 only This will reduce lags and provide a fully
immersive gaming experience.
Answer: B
Explanation:
63. Consider the following statements with
Statement 1 is incorrect: The main goal of edge reference to Internet of Things:
computing is to reduce latency requirements while
1. Internet of Things is the inter-linking
processing data and saving network costs.
of digital devices, machines and even
Statement 2 is correct: One of the best ways to people with each other through
implement edge computing is in smart home wireless networks.
devices.
2. IoT has use in the field of Agriculture
Statement 3 is correct: It also has use in the cloud too
gaming industry.
Which of the statements given above is/are
Additional info: correct?
● Edge computing is a distributed IT A. 1 only
architecture which moves computing
B. 2 only
resources from clouds and data centers as
close as possible to the originating source. C. Both 1 and 2

● The main goal of edge computing is to D. Neither 1 nor 2


reduce latency requirements while Answer: C
processing data and saving network costs.
Explanation:
Examples and Use Cases
Statement 1 is correct: Internet of Things is the
● One of the best ways to implement edge inter-linking of digital devices, machines, people
computing is in smart home devices. In and other objects with each other through wireless
smart homes, a number of IoT devices networks. It enables anything and everything to
collect data from around the house. connect and communicate with each other. It is the
● The data is then sent to a remote server future internet. Because the first version of the
where it is stored and processed. This internet was about data created by the internet. The
architecture can cause a number of next version is about the data created by things.
problems in the event of a network outage. Statement 2 is correct: IoT can be utilized to collect
data about rainfall, soil moisture, soil nutrients,
pest infestation etc. It can assist in making A. 1 only
informed decisions to increase agricultural B. 2 and 3 only
production as well as reducing the risks of crop
failures etc. It can help make agriculture profitable C. 3 only
with better price-discovery for farmers through D. 1 and 2 only
smart techniques.
Answer: D
Additional info:
Explanation:
● Internet of Things (IoT) refers to the
Statement 1 is correct: Artificial Intelligence is the
interconnection of various devices, sensors,
concept of creating smart intelligent machines.
and machines through the internet,
allowing them to share data and work Statement 2 is correct: Machine Learning is a
subset of artificial intelligence that helps you build
● together seamlessly. The IoT market has
AI-driven applications.
been rapidly expanding in recent years,
with projections suggesting that there will Statement 3 is incorrect: Deep Learning is a subset
be over 41 billion connected devices by of machine learning that uses vast volumes of data
2027. and complex algorithms to train a model.

● The potential applications of IoT are vast Additional info:


and span across various sectors such as ● Artificial intelligence (AI) is the field of
healthcare, manufacturing, transportation, computer science that aims to create
agriculture, and smart cities. IoT can enable machines that can perform tasks requiring
real-time monitoring of equipment and human-level intelligence, such as
machines, leading to reduced downtime reasoning, perception, decision-making,
and increased efficiency. It can also and natural language understanding.
facilitate remote
● Machine learning is a subset of AI that
focuses on developing algorithms that
64. Consider the following statements: allow machines to learn from data without
explicit programming.
1. Artificial Intelligence is the concept of
creating smart intelligent machines. ● Deep learning is an advanced form of
machine learning that involves building
2. Machine Learning is a subset of
artificial neural networks with multiple
artificial intelligence that helps you
layers to analyze and process complex data
build AI-driven applications.
such as images, audio, and text.
3. Deep Learning is a subset of artificial
● Deep learning has revolutionized the field
intelligence that uses vast volumes of
of AI by enabling machines to perform
data and complex algorithms to train a
tasks such as image and speech
model.
recognition, natural language processing,
Which of the statements given above is/are and autonomous driving with
correct? unprecedented accuracy, speed, and
efficiency.
● AI, machine learning, and deep learning
are rapidly transforming various 66. Which of the following is/are challenges for
industries, including healthcare, finance, 5G Rollout in India?
transportation, and manufacturing, and are
1. Low Fiberization Footprint
expected to have a significant impact on
our economy and society in the coming 2. ‘Make in India’ Hardware Challenge
years. 3. High Spectrum Pricing
Select the correct code:
65. Which of the following statements is incorrect A. 1 and 2 only
with reference to evolution of
Telecommunication? B. 2 and 3 only

A. 1G worked on analog radio signals C. 1 and 3 only

B. 2G uses digital radio signals and D. 1,2 and 3


supports both voice and data Answer: D
transmission.
Explanation:
C. 3G had the ability to transmit
Challenges for 5G Rollout in India:
telephone signals including digitised
voice, video calls and conferencing. ● Low Fiberization Footprint: There is a need
to upgrade fibre connectivity across India,
D. 4G had a peak speed of 100 Mbps to 1
which at present connects only 30% of
Gbps but did not support 3D virtual
India’s telecom towers.
reality.
○ For an efficient 5G India launch and
Answer: D
adoption, this number has to
Explanation: double.
Statement 1 is correct: 1G was launched in the ● ‘Make in India’ Hardware Challenge: The
1980s and worked on analog radio signals and ban on certain foreign telecom OEMs
supported only voice calls. (original equipment manufacturer) upon
Statement 2 is correct: 2G was launched in the which most of the 5G technology
1990s which uses digital radio signals and supports development depends, presents a hurdle in
both voice and data transmission with a itself.
bandwidth of 64 Kbps. ● High Spectrum Pricing: India’s 5G
Statement 3 is correct: 3G was launched in the spectrum pricing is several times costlier
2000s with a speed of 1 Mbps to 2 Mbps and it has than the global average.
the ability to transmit telephone signals including ○ This will be of detriment to India’s
digitised voice, video calls and conferencing. cash-strapped telcos.
Statement 4 is incorrect: 4G was launched in 2009 ● Choosing the Optimal 5G Technology
with a peak speed of 100 Mbps to 1 Gbps and it also Standard: The tussle between the
enables 3D virtual reality. homegrown 5Gi standard and the global
3GPP standard needs to be concluded in offers the potential for economic growth
order to hasten 5G technology and improved quality of life for citizens.
implementation.
○ While 5Gi brings obvious benefits, 67. Consider the following statements with
it also increases 5G India launch reference to International Classification of
costs and interoperability issues for Diseases (ICD):
telcos.
1. It is the global health information
Additional info: standard for mortality and morbidity
● 5G is the fifth generation of mobile statistics.
networks that offers faster and more 2. Allocation of resources is also one of
reliable connectivity than its predecessors. the tasks of ICD
● It has the potential to revolutionize 3. Recently, Gaming disorder has been
industries such as healthcare, added to the section on addictive
transportation, and manufacturing by disorders.
enabling the Internet of Things (IoT) and
Which of the statements given above is/are
machine-to-machine communication.
correct?
● 5G networks use high-frequency radio
A. 1 and 2 only
waves that have shorter wavelengths than
previous generations, allowing for more B. 2 and 3 only
data to be transmitted at faster speeds. C. 1 and 3 only
● The technology also has lower latency, D. 1, 2 and 3
meaning that data can be sent and received
Answer: D
almost instantaneously.
Explanation:
● 5G networks require more infrastructure
than previous generations, including more Statement 1 is correct: It is the global health
cell towers and fiber-optic cables to support information standard for mortality and morbidity
the increased data transmission. statistics.

● The deployment of 5G networks is Statement 2 is correct: Allocation of resources is


currently underway, with many countries also one of the tasks of ICD. ICD is increasingly
already rolling out the technology in major used in clinical care and research to define diseases
cities. and study disease patterns, as well as manage
health care, monitor outcomes and allocate
● However, there are concerns over the
resources.
potential health risks associated with the
increased exposure to radio waves, as well Statement 3 is correct: Recently, Gaming disorder
as the security risks posed by the use of 5G has been added to the section on addictive
networks in critical infrastructure. disorders.

● The adoption of 5G technology is a priority Additional info:


for many countries, including India, as it
What is ICD? Explanation:
The ICD is the global health information standard Statement 1 is correct: Viruses do not grow,
for mortality and morbidity statistics. The first neither respire nor metabolize, but they reproduce.
international classification edition, known as the Statement 2 is incorrect: The genetic material of
International List of Causes of Death, was adopted viruses can be composed of DNA or RNA. A virus
by the International Statistical Institute in 1893. is an infectious microbe consisting of a segment of
WHO was entrusted with the ICD at its creation in nucleic acid (either DNA or RNA) surrounded by
1948. The ICD is revised periodically and is a protein coat.
currently in its 10th revision.
Statement 3 is correct: Viruses are surrounded by
● ICD is increasingly used in clinical care and
a protein coat
research to define diseases and study
disease patterns, as well as manage health Statement 4 is correct: Viruses are considered both
care, monitor outcomes and allocate as living and non-living things
resources. ICD has been translated into 43 Additional info:
languages.
● A virus is an infectious microbe consisting
● More than 100 countries use the system to of a segment of nucleic acid (either DNA or
report mortality data, a primary indicator RNA) surrounded by a protein coat.
of health status. This system helps to
● A virus cannot replicate alone; instead, it
monitor death and disease rates worldwide
must infect cells and use components of the
and measure progress towards the
host cell to make copies of itself.
Millennium Development Goals.
● Often, a virus ends up killing the host cell
● About 70% of the world’s health
in the process, causing damage to the host
expenditures (USD $ 3.5 billion) are
organism.
allocated using ICD for reimbursement and
resource allocation. ● Well-known examples of viruses causing
human disease include AIDS, COVID-19,
measles and smallpox.
68. Which of the following is not a property of
viruses?
A. Viruses do not grow, neither respire
nor metabolize, but they reproduce.
B. The genetic material of viruses is
composed of DNA only
C. Viruses are surrounded by a protein
coat
D. Viruses are considered both as living
and non-living things
69. Variants of Concern do not have which of the
Answer: B following characteristics?
A. Higher transmissibility ● Specific genetic markers that are
predicted to affect therapeutics,
B. Change in virulence/disease
transmission, diagnostics or
presentation; evidence of increased
immune escape.
disease severity
● Evidence that it is the cause of an
C. Reduced susceptibility to treatment
increased proportion of cases or
D. Evading diagnostics, drugs and unique outbreak clusters.
vaccines
● Variant of High Consequence: A variant
Answer: C of high consequence has clear evidence that
Explanation: prevention measures or medical
Variant of Concern (VOC): When there is evidence countermeasures (MCMs) have drastically
for increased transmissions through field and reduced effectiveness relative to previously
clinical investigations, a variant becomes a VOC. circulating variants. Possible attributes
Variants of Concern have one or more of the include:
following characteristics. ● Demonstrated diagnostics failure
● Higher transmissibility ● Higher severity of the disease and
● Change in virulence/disease presentation; increased hospitalisations
evidence of increased disease severity ● Reduced susceptibility to treatment
● Evading diagnostics, drugs and vaccines ● Evidence for decreased vaccine
Additional info: effectiveness, a very large number
of vaccine breakthrough cases, or
● Variant under Investigation: A variant very low protection from vaccine
becomes a variant under investigation against severe disease
when the mutations happen and if there is
any previous association with any other
similar variant which is felt to have an 70. Consider the following statements with
impact on public health. reference to medical oxygen:
● Variant of Interest (VOI): This includes 1. There are traces of nitrogen and argon
variants with specific genetic markers that in oxygen concentrators
have been associated with changes to 2. There are no known side effects of
receptor binding, decreased neutralization medical oxygen
by antibodies generated against
3. The National Pharmaceutical Pricing
vaccination/previous infection, lessened
Authority controls and monitors the
efficacy of treatments, potential diagnostic
medical oxygen prices in India.
impact, or predicted increase in
transmissibility or disease severity. The Which of the statements given above is/are
following are the possible attributes of a correct?
VOI. A. 1 only
B. 2 and 3 only cause severe damage to the lungs and other
organ systems.
C. 2 only
D. 1 and 3 only
71. Consider the following statements with
Answer: D
reference to types of corona vaccines:
Explanation:
1. Pfizer and Moderna vaccines developed in
Statement 1 is correct: Medical oxygen comprises the US are mRNA vaccines.
minimum 90% oxygen with 5% nitrogen and 5%
2. Johnson & Johnson, Sputnik V and
argon. Mostly, oxygen concentrators or oxygen
AstraZeneca-Oxford University vaccine is
plants contain 90-95% O2 with traces of nitrogen
a kind of viral vector vaccine.
and argon.
3. Bharat Biotech’s Covaxin and Sinovac-
Statement 2 is incorrect: There are side effects of
CoronaVac is categorized as an inactivated
medical oxygen
vaccine.
Statement 3 is correct: National Pharmaceutical
4. Corbevax is a recombinant protein subunit
Pricing Authority controls and monitors the
vaccine.
medical oxygen prices in India.
Which of the statements given above is/are
Additional info:
correct?
Side effects of medical oxygen: If it is used above
A. 1 and 2 only
the prescribed amount, the medical oxygen creates
certain side effects. This is why the doctor's B. 3 only
prescription of oxygen is essential. The side effects C. 1, 2 and 4 only
include,
D. 1, 2, 3 and 4
● Convulsions or seizures will appear in
Answer: D
patients after a few hours of exposure to
oxygen at pressures above 3 bar(g). Explanation:

● It can cause Retrolenticular fibroplasia in Statement 1 is correct: Pfizer and Moderna


premature infants if they are exposed to vaccines developed in the US are mRNA vaccines.
oxygen concentrations greater than 40%. In Statement 2 is correct: Johnson & Johnson, Sputnik
short, it is a condition of abnormal growth V and AstraZeneca-Oxford University vaccine is a
of blood vessels in the eye. Retrolenticular kind of viral vector vaccine.
fibroplasia is the leading cause of child
Statement 3 is correct: Bharat Biotech’s Covaxin
blindness today in the world.
and Sinovac-CoronaVac is categorised as an
● Some patients will also develop coughing inactivated vaccine.
and breathing difficulties after they are put
Statement 4 is correct: Corbevax is a recombinant
under medical oxygen.
protein subunit vaccine.
● The issue of Oxygen toxicity: Excessive or
Additional info:
inappropriate supplemental oxygen can
Different types of Covid-19 vaccines:
Vaccines are categorized on the basis of the ● Spike Protein Vaccines:
development process adopted by scientists. ○ Corbevax is a recombinant protein
● mRNA Vaccines: subunit vaccine.
○ Pfizer and Moderna vaccines ○ The vaccine is being indigenously
developed in the US are mRNA produced by Biological E,
vaccines. Hyderabad-based Pharmaceutical
company.
○ In this type, vaccines will carry the
molecular instructions to make the ○ The Corbevax vaccine is made up of
protein in the body through a a specific part of SARS-CoV-2 — the
synthetic RNA of the virus. spike protein on the virus’s surface.
○ The host body uses this to produce ○ The members of the coronavirus
the viral protein that is recognized family have sharp bumps that
and thereby making the body protrude from the surface of their
mount an immune response against outer envelopes. Those bumps are
the disease. known as spike proteins.
● Viral Vector Vaccines: ○ These spike proteins allow the virus
○ Johnson & Johnson, Sputnik V and to enter the cells in the body so that
AstraZeneca-Oxford University it can replicate and cause disease.
vaccine is a kind of viral vector ○ However, in Corbevax Vaccine the
vaccine. spike protein alone is given to the
○ In this type, a modified version of body. So, it is not expected to be
other viruses (for example, harmful as the rest of the virus is
adenovirus) is used. absent. The body is expected to
develop an immune response
○ The virus can enter human cells but against the injected spike protein.
not replicate inside.
○ Therefore, when the real virus
○ A gene for the coronavirus vaccine attempts to infect the body, it will
was added into the adenovirus already have an immune response
DNA, allowing the vaccine to target ready that will make it unlikely for
the spike proteins that SARS-CoV-2 the person to fall severely ill.
uses to enter human cells.
● Inactivated Vaccines:
72. Consider the following statements with
○ Bharat Biotech’s Covaxin and reference to difference between DNA and RNA
Sinovac-CoronaVac is categorised Vaccines:
as an inactivated vaccine. In this
1. DNA vaccine is delivered through
type, an inactivated live virus is
plasmid vectors
used to create an immune response
against the disease. 2. RNA Vaccines are highly stable for
storage and shipping
They might integrate They do not enter the
3. RNA Vaccines do not enter the cell with the natural cell nucleus and
nucleus and integrate with the host genome of the host. integrate with the
genome host genome.
Which of the statements given above is/are
correct?
73. Which of the following statements is incorrect
A. 1 and 2 only
with reference to T cells?
B. 2 and 3 only
A. T cells mature and undergo positive
C. 1 and 3 only and negative selection processes in the
D. 1, 2 and 3 thymus.

Answer: C B. T cells play a vital role in cell-mediated


and humoral immunity.
Explanation:
C. T cells can recognize soluble, free
Statement 1 is correct: DNA vaccine is delivered
antigens.
through plasmid vectors
D. T cells can also recognize protein-
Statement 2 is incorrect: DNA Vaccines are highly
based, receptor-bound antigens.
stable for storage and shipping
Answer: C
Statement 3 is correct: RNA Vaccines do not enter
the cell nucleus and integrate with the host genome Explanation:

Additional info: Statement 1 is correct: T cells are a diverse and


important group of lymphocytes that mature and
Difference between DNA and RNA Vaccines
undergo positive and negative selection processes
DNA Vaccines RNA Vaccines in the thymus.
Statement 2 is correct: T cells play a vital role in
both components of active immunity, including
This vaccine This vaccine cell-mediated and to some extent humoral
transfects specific transfects mRNA to immunity.
DNA to create an create an immune Statement 3 is incorrect: T cells cannot recognize
immune response. response. soluble, free antigens.

DNA vaccine is RNA vaccine is Statement 4 is correct: T cells can only recognize
delivered through delivered through protein-based, receptor-bound antigens.
plasmid vectors. lipid nanoparticles. Additional info:
● T cells are a diverse and important group of
It is highly stable for It is comparatively lymphocytes that mature and undergo
storage and shipping. less stable. positive and negative selection processes in
the thymus.
● Types: enormous amounts of data and process it in
○ There are several types of T cells; real-time to solve critical problems.
the most common and well-known ● In recent years, the demand for
are the CD4+ T cells (helper T cells) supercomputers has been steadily rising
and CD8+ T Cells (cytotoxic T cells, across various industries, including
or killer T cells). aerospace, defense, finance, healthcare, and
○ As the names suggest, helper T cells weather forecasting. Supercomputers are
‘help’ other cells of the immune being used to simulate complex weather
system, whilst cytotoxic T cells kill patterns, predict stock market trends,
virally infected cells and tumors. design new drugs and vaccines, conduct
advanced research in fields such as
astrophysics and climate science, and even
74. Supercomputers are used in which of the create realistic virtual environments for
following fields? gaming and entertainment purposes.
1. Weather forecasting ● In India, the government has been actively
2. Intelligence gathering and analysis promoting the development and adoption
of supercomputers to support research and
3. Data mining
innovation. The National Supercomputing
Select the correct code: Mission (NSM) is a prime example of this
effort, aimed at providing access to high-
A. 1 and 2 only
performance computing infrastructure to
B. 2 and 3 only scientists, researchers, and startups across
C. 1 and 3 only the country. Under the NSM, several
supercomputers have been installed in
D. 1, 2 and 3
various institutes and universities,
Answer: D enabling scientists to conduct cutting-edge
research in areas such as genomics, particle
● Supercomputers are primarily designed to
physics, and climate modeling.
be used in enterprises and organizations
that require massive computing power.
○ For example: weather forecasting, 75. Consider the following statements with
scientific research, intelligence reference to Quantum Key Distribution
gathering and analysis, data mining Technology:
etc. 1. It is a mechanism to develop secure
Additional info: communication.
● Supercomputers refer to high-performance 2. QKD is also used to transmit any
computing systems that can perform
message data
extremely complex and sophisticated
computations at a lightning-fast speed. 3. The distribution of encryption keys is
These machines are designed to handle the crucial factor for QKD technology.
Which of the statements given above is/are 75. Consider the following statements with
correct? reference to LiDAR Technology:
A. 1 only 1. LiDAR uses light across ultraviolet and
B. 2 and 3 only visible only
C. 1 and 3 only 2. LiDAR can be easily used in dense
D. 2 only forests and thick vegetation.

Answer: C 3. It is also used for calculating


phytoplankton fluorescence
Explanation:
Statement 1 is correct: It is a mechanism to develop Which of the statements given above is/are
secure communication. correct?

Statement 2 is incorrect: Quantum key A. 1 only


distribution is only used to produce and distribute B. 2 and 3 only
a key, not to transmit any message data. This key
C. 3 only
can then be used with any chosen encryption
algorithm to encrypt (and decrypt) a message, D. 1 and 2 only
which can then be transmitted over a standard
Answer: C
communication channel.
Statement 1 is incorrect: LiDAR uses light across
Statement 3 is correct: The distribution of
different wavelengths including ultraviolet,
encryption keys is the crucial factor for QKD
visible, or near-infrared light to image objects and
technology.
it’s, as such, able to detect all kinds of material
Additional info: compositions, including non-metals, rocks, rain,
What is Quantum Key Distribution Technology? chemical compounds, aerosols, clouds and even
single molecules.
● QKD, also called Quantum Cryptography,
is a mechanism to develop secure Statement 2 is incorrect: LiDAR is not suitable to
communication. be used in dense forests and thick vegetation.

● It provides a way of distributing and Statement 3 is correct: It is used for calculating


sharing secret keys that are necessary for phytoplankton fluorescence and biomass in the
cryptographic protocols. surface of the ocean.

● The conventional cryptosystems used for Additional info:


data-encryption rely on the complexity of What is LiDAR?
mathematical algorithms, whereas the
LiDAR stands for Light Detection and Ranging. It
security offered by quantum
is a ranging technology in which the distance of an
communication is based on the laws of
object is measured by firing beams of light at the
Physics.
object and using the time as well as the wavelength
of the reflected beam of light to estimate the
distance. These light pulses – when combined with
other data help in generating high-resolution, What is 3D Printing?
accurate 3D information of the object. ● 3D printing uses computer-aided design
(CAD) to create three-dimensional objects
through a layering method.
76. Which of the following statements is incorrect
with reference to 3D printing? Principle
A. A 3D printer makes a three-dimensional ● In 3D printing, a 3D printer makes a three-
object from a CAD (computer-aided dimensional object from a CAD (computer-
design) file. aided design) file.
B. 3D printing uses liquid ink for the purpose. ● The creation of a 3D printed object is
achieved using additive processes.
C. 3D printing can manufacture parts within
hours, which speeds up the prototyping ● In an additive process an object is created
process. by laying down successive layers of
material until the object is created.
D. 3D printing is also being used in the
medical sector ● Each of these layers can be seen as a thinly
Answer: B sliced cross-section of the object.

Statement 1 is correct: A 3D printer makes a three- ● 3D printing enables us to produce complex


dimensional object from a CAD (computer-aided shapes using less material than traditional
design) file. manufacturing methods.

Statement 2 is incorrect: Where an inkjet printer


sprays liquid ink and a laser printer uses solid 77. Which of the following is India's first
powder, a 3D printer uses neither. indigenous server?
● The 3-D printer deposits layers of molten A. Trinetra
plastic or powder and fuses them together B. Rudra
(and to the existing structure) with
adhesive or ultraviolet light. C. Param

The most common 3D printing raw materials are D. Shivay


the commodity thermoplastic polymers: Answer: B
Acrylonitrile butadiene styrene (ABS), Polylactic Explanation:
acid (PLA), Polyethylene terephthalate glycol-
modified (PETG). The Centre for Development of Advanced
Computing (C-DAC), under the Ministry of
Statement 3 is correct: 3D printing can Electronics and Information Technology (MeitY),
manufacture parts within hours, which speeds up has designed and developed the Rudra I server
the prototyping process. towards its efforts to make a supercomputer.
Statement 4 is correct: 3D printing is also being
used in the medical sector.
78. RADAR has applications and uses in which of
Additional info: the following?
1. Law enforcement control for monitoring commercial flights.
It is also used in automotive safety features
2. Space
such as collision avoidance systems.
3. Remote sensing of environment
4. Aircraft navigation
79. Under Drone Regulation 1.0, which of the
5. Air Traffic Controller following are restricted locations for flying the
Select the correct code: Drones?

A. 1 and 2 only 1. Airports

B. 3 and 4 only 2. Near international border

C. 1, 3 and 5 only 3. Near coast line

D. 1, 2, 3, 4 and 5 4. Military installations

Answer: D Select the correct code:

Explanation: A. 1 and 2 only

Applications and uses of Radar are given below: B. 4 only

● Military. C. 3 and 4 only

● Law enforcement. D. 1, 2, 3 and 4

● Space. Answer: D

● Remote sensing of environment. Explanation:

● Aircraft navigation. As per Drone Regulation 1.0 issued by the Ministry


of Civil Aviation in India in 2018, there are a few
● Ship Navigation.
restrictions on flying drones in certain locations.
● Air Traffic Controller. These restricted locations include:
Additional info: • Within 5 km of an airport or airport
About Radar: boundary
• Within 25 km from international
● Radar, which stands for Radio Detection
borders, including the Line of Control
and Ranging, is a technique used to detect
(LoC), Line of Actual Control (LAC),
and locate objects through the use of radio
and Actual Ground Position Line
waves. It works by transmitting radio
(AGPL)
waves from an antenna and then
• Within 500 m of a military installation
measuring the reflected waves that bounce
or strategic location
back off a target object.
• Within 50 km from the coast-line
● Radar technology has many applications • Temporary no-fly zones, as declared
including in the military for detecting by the local administration or the
enemy aircraft, in weather forecasting for Ministry of Home Affairs.
measuring precipitation, and in air traffic
Additional information:
Drone Regulation 1.0 Explanation:
• Under this regulation, the Digital Sky Statement 1 is incorrect: All civilian drone
Platform will enable online operations will be restricted to only during day
registration of pilots, devices, service time and a maximum of 400 feet altitude.
providers, and NPNT (no permission, Statement 2 is correct: There can’t be any human
no take-off). or animal payloads, or anything hazardous.
• The Digital Sky Platform is a unique
unmanned traffic management (UTM) Statement 3 is correct: It cannot in any manner
system which is expected to facilitate cause danger to people or property and insurance
registration and licensing of drones will be mandatory to cover third-party damage.
and operators in addition to giving Statement 4 is correct: Except nano drones and
instant (online) clearances to operators those owned by National Technical Research
for every flight. Organisation and the central agencies, the rest
• The airspace has been partitioned into would be registered and issued a Unique
Red Zone (flying not permitted), Identification Number.
Yellow Zone (controlled airspace), and
Green Zone (automatic permission).
The restricted locations are airports, 81. Which of the following is a Disadvantages of
near international border, near coast Robotics?
line, state secretariat complexes, A. In many situations robots can increase
strategic locations, military productivity, efficiency, quality and
installations. consistency of products.
B. Robots can work in environments
which are unsafe for humans as they
80. Which of the following statements with don’t have the same environmental
reference to Drone is incorrect?
requirements that humans do – such as
A. There is no restriction on civilian drone lighting, air conditioning or noise
operations. protection.
B. There can’t be any human or animal C. Robots have some sensors/actuators
payloads, or anything hazardous. which are more capable than humans.
C. It cannot in any manner cause danger D. Robots can only do what they are told
to people or property and insurance to do.
will be mandatory to cover third-party
Answer: D
damage.
D. Except nano drones and those owned Explanation:
by National Technical Research Statement 4 is correct: Robots can only do what
Organisation and the central agencies, they are told to do – they can’t improvise is a
the rest would be registered and issued disadvantage of robotics.
a Unique Identification Number.
Disadvantages:
Answer: A
• The use of robots can create economic Development Organisation for the Indian
problems if they replace human jobs. Air Force and Indian Navy. The UCAV will
be capable of releasing missiles, bombs and
• Robots can only do what they are told to do
precision-guided munitions.
– they can’t improvise
• This means that safety procedures are
needed to protect humans and other robots. 83. Consider the following statements:
• Although robots can be superior to humans 1. Chaturobot are Vision sensors to pick
in some ways, they are less dexterous than objects
humans. 2. smartNAV is a robot for navigating the
o Robotics lack emotional moon surface in the next manned mission
intelligence, which plays a critical (ISRO)
role in intense situations. Which of the statements given above is/are
• Often robots are very costly – in terms of correct?
the initial cost, maintenance, the need for A. 1 only
extra components and the need to be
B. 2 only
programmed to do the task.
C. Both 1 and 2
• Surveillance concerns pose a problem of
entering a privacy nightmare. D. Neither 1 nor 2

Answer: C
82. Which of the following statements is incorrect Explanation:
with reference to AURA?
Statement 1 is correct: Chaturobot: Vision sensors
A. It is an autonomous unmanned combat to pick objects (DRDO+CAIR.) Centre for Artificial
air vehicle (UCAV) Intelligence and Robotics (CAIR) is the premier
B. It is being developed by the Defence laboratory for R&D in different areas in
Research and Development Information and Communication Technology
Organisation for the Indian Air Force (ICT) as applicable to Defence.
and Indian Navy.
Statement 2 is correct: smartNAV is a robot for
C. The UCAV will be capable of releasing
navigating the moon surface in the next manned
missiles, bombs and precision-guided
mission (ISRO)
munitions.
D. None of the above
Answer: D 84. As per Unmanned Aircraft System (UAS)
Rules, 2020 consider the following:
Explanation:
1. Only entities authorized by the
All the statements are correct.
Directorate General of Civil Aviation
● AURA is an autonomous unmanned can own or operate drones.
combat air vehicle (UCAV), being
developed by the Defence Research and
2. No drone can be operated in India B. 3 and 4 only
unless there is the existence of a valid C. 2 and 4 only
third-party insurance policy.
3. For owning and using a drone, one has D. 1, 2 and 3
to be at least 21 years of age. Answer: D
Which of the above statement(s) is/are correct? Explanation:
A. 1 and 2 only A disease is a condition that is marked by 3 basic
B. 2 and 3 only factors.

C. 1 and 3 only ● An established biological cause behind the


condition
D. 1, 2 and 3
● A defined group of symptoms
Answer: A
● Consistent change in anatomy due to the
Explanation: condition.
Statement 1 is correct: Only entities authorized by
the Directorate General of Civil Aviation can own
86. Antibiotics can be used to treat which of the
or operate drones.
following?
Statement 2 is correct: No drone can be operated
1. Strep throat
in India unless there is the existence of a valid third
party insurance policy. 2. Whooping cough

Statement 3 is incorrect: The minimum age for 3. Flu


using and owning a drone is 18 years and not 21 4. Urinary tract infection (UTI)
years.
Select the correct code:
A. 1 and 2 only
85. A disease is a condition marked by which of
B. 3 only
the following basic factors?
C. 1, 2 and 4 only
1. An established biological cause behind
the condition D. 2, 3 and 4 only

2. A defined group of symptoms Answer: C

3. Consistent change in anatomy due to Explanation:


the condition. Antibiotics ONLY treat certain infections caused
4. Leading to feelings of energy depletion by bacteria, such as:
or exhaustion ● Strep throat
Which of the statements given above is/are ● Whooping cough
correct?
● Urinary tract infection (UTI)
A. 1 only
Antibiotics are also needed to treat life-threatening ● It refers to the amount of genetic material,
conditions caused by bacteria, such as sepsis, commonly RNA, of a virus present in an
which is the body’s extreme response to infection. infected person’s blood.
Antibiotics DO NOT work on viruses, such as ● This is expressed as the total number of
those that cause: viral particles present in each milliliter of
blood.
● Colds and runny noses, even if the mucus
is thick, yellow, or green ● A higher viral load in the blood means that
the virus is replicating and the infection is
● Most sore throats (except strep throat)
progressing.
● Flu
● An infected person with a high viral load is
● Most cases of chest colds (bronchitis) more likely to shed more virus particles, in
the process known as “viral shedding”.
87. Consider the following statements: ● Ct Value:
1. Viral Load refers to the amount of ○ Ct is short for ‘Cycle Threshold’.
genetic material of a virus present in an
○ The Ct value refers to the number of
infected person’s blood.
cycles after which the virus can be
2. Severity of the disease has a stronger detected.
relationship with Ct values
Which of the statements given above is/are
correct?
A. 1 only
B. 2 only
C. Both 1 and 2
D. Neither 1 nor 2
Answer: A
Explanation: ○ If a higher number of cycles is
required, it implies that the virus
Statement 1 is correct: Viral Load refers to the
went undetected when the number
amount of genetic material of a virus present in an
of cycles was lower.
infected person’s blood.
○ The lower the Ct value, the higher
Statement 2 is incorrect: The time since the onset
the viral load-because the virus has
of symptoms has a stronger relationship with Ct
been spotted after fewer cycles.
values as compared to the severity of the disease.
○ It has been found that the time since
Additional info:
the onset of symptoms has a
What is Viral Load? stronger relationship with Ct
values as compared to the severity resistance to a disease
of the disease. 3. Immunization is the process of making
one resistant to an infectious disease
88. Greek alphabets were used as labels for Covid usually through vaccination.
strains. Consider the following pairs of Greek 4. Complete immunity does not occur
alphabets and the countries where the earliest even when the person fully recovers
samples were detected: from the disease.
1. Alpha- United Kingdom Which of the statements is/are correct?
2. Beta- Brazil A. 1 and 2 only
3. Gamma- South Africa B. 3 and 4 only
4. Delta- India C. 2 and 4 only
Which of the pairs is correctly matched? D. 1 and 3 only
A. 1 and 2 only Answer: D
B. 3 and 4 only Explanation:
C. 2 and 3 only Statement 1 is correct: Vaccination is usually
D. 1 and 4 only injected or administered orally

Answer: D Statement 2 is incorrect: Vaccination does not


guarantee complete resistance to a disease
Explanation:
Statement 3 is correct: Immunization is the process
● Alpha: VOC B.1.1.7, samples earliest
of making one resistant to an infectious disease
documented in United Kingdom
usually through vaccination.
(September 2020)
Statement 4 is incorrect: Complete immunity
● Beta: VOC B.1.351, samples earliest
occurs when the person fully recovers from the
documented in South Africa (May 2020)
disease
● Gamma: VOC P.1, samples earliest
Additional info:
documented in Brazil (November 2020)
Difference Between Vaccination and
● Delta: B.1.617.2, samples earliest
Immunization.
documented in India (October 2020)
Vaccination Immunization

89. Consider the following statements with The process involves The process of making
reference to difference Between Vaccination and using vaccines to one resistant to an
Immunization: trigger an immune infectious disease
1. Vaccination is usually injected or response to protect usually through
administered orally against vaccination.
infections/diseases.
2. Vaccination guarantees complete
● Supercomputers have become important
It is usually injected It is not administered
tools for research and development
or administered in any way. The body
activities in various fields like aerospace,
orally. develops resistance
climate modelling, molecular modelling,
from vaccines.
sequencing etc. In India, the use of
Imovax Rabies is the The body builds up supercomputers has been increasing
trade name for rabies immunity through this rapidly with the launch of various
vaccine. vaccine for the disease initiatives by the government in the last
rabies. few years. This article will discuss various
aspects of supercomputers in India and
Vaccination does not Complete immunity their role in the development of the nation.
guarantee complete occurs when the
● The National Supercomputing Mission
resistance to a person fully recovers
(NSM) launched by the government in 2015
disease. from the disease.
aims to develop the supercomputing
Usually, if mutation Similarly, variations of infrastructure of the country and make it
happens to microbes, a disease impact the available to the scientific and research
it might render the body’s ability to community at affordable prices. The NSM
vaccine ineffective generate an immune is a joint initiative of the Department of
(this is the reason response. Science and Technology and Department of
why common cold Electronics and Information Technology.
has no vaccine). ● The first supercomputer in India was
PARAM 8000, developed by the Centre for
Development of Advanced Computing (C-
90. Which of the following countries has the DAC) in Pune. Since then, India has made
maximum number of supercomputers? significant progress in the field of
A. US supercomputers. The two most powerful
supercomputers in the country are Param-
B. China
Siddhi-AI and Mihir in India. Param-
C. Japan Siddhi-AI, launched in September 2020, is
D. France India’s fastest and AI-enabled
supercomputer with a peak computing
Answer: B power of 5.267 petaflops, while Mihir can
Explanation: provide a peak computing power of 1.5
petaflops.
Globally, China has the maximum number of
supercomputers and maintains the top position in ● Many government and research
the world, followed by the US, Japan, France, organizations in India have their own
Germany, Netherlands, Ireland and the United supercomputing facilities, including Indian
Kingdom. Institute of Technology (IIT), Indian Space
Research Organization (ISRO), Council of
Additional info:
Scientific and Industrial Research (CSIR),
Supercomputers in India:
Defence Research and Development Statement 2 is incorrect: Dark net is a network
Organization (DRDO) and others. These built over the Internet
facilities not only support the research Statement 3 is incorrect: Dark web refers to
activities in India but also enable websites on a darknet.
collaborations with international research
teams. Additional info:

● Supercomputers in India have been used Dark Net, Deep Web and Surface Web
for various purposes like the development ● "Dark net" is commonly confused with
of COVID-19 vaccines, weather forecasting, "deep web." The deep web refers to
climate modelling, drug discovery, unindexed sites which are unsearchable; in
genomic research etc. For instance, the most cases, this is because those sites are
supercomputer facility at IIT-Kharagpur is protected by passwords.
being used to develop an artificial ● Part of the WWW (World Wide Web)
intelligence-based system that can predict which is not indexed by a search engine like
the level of pollution in the air using Google is the Deep Web and it is about 500-
satellite images. It has also been used to 600 times larger than the surface web.
develop a tool that can predict the spread
of the coronavirus in different regions of ● Surface Web -Also called the Visible Web,
the country. Indexed Web, Indexable Web or Lightnet -
is that portion of the World Wide Web that
is readily available to the general public
91. Consider the following statements: and searchable with standard web search
1. Surface Web only constitutes 4-6% of the engines. It is the opposite of the deep web.
whole web. It only constitutes 4-6% of the whole web.

2. Dark web is a network built over the


Internet 92. Which of the following is/are advantages of
Optical Fibre Communication?
3. Darknet refers to websites on the dark web.
1. Economical and cost-effective
Which of the statements given above is/are
correct? 2. Thin and non-flammable
A. 1 only 3. Less power consumption
B. 2 and 3 only 4. Less signal degradation
C. 1 and 3 only 5. Flexible and lightweight
D. 2 only Select the correct code:
Answer: A A. 1 and 2 only
Explanation: B. 2, 4 and 5 only
Statement 1 is correct: Surface Web only C. 3 and 5 only
constitutes 4-6% of the whole web.
D. 1, 2, 3, 4 and 5
Answer: D D. 1 and 2 only
Explanation: Answer: D
What is an Optical Fibre? Statement 1 is correct: The aim of the project is to
● Optical fibre is the technology associated connect gram panchayats through broadband
with data transmission using light pulses Statement 2 is correct: It is a flagship mission
traveling along with a long fibre which is implemented by Bharat Broadband Network Ltd.
usually made of plastic or glass. Metal (BBNL).
wires are preferred for transmission in
Statement 3 is incorrect: There is a delay in the
optical fibre communication as signals
Bharatnet project, aimed at connecting gram
travel with fewer damages. Optical fibres
panchayats through broadband, which has
are also unaffected by electromagnetic
resulted in a cost escalation from Rs 20,100 crore to
interference.
Rs 61,109 crore. This is partly due to the
● The fibre optical cable uses the application ineffectiveness of Bharat Broadband Network
of total internal reflection of light. The (BBNL), the entity set up to implement the project
fibres are designed such that they facilitate to connect 250,000 gram panchayats with optic
the propagation of light along with the fibre, for high-speed broadband connectivity. With
optical fibre depending on the requirement BBNL not delivering the desired results, the
of power and distance of transmission. government now plans to merge BBNL with BSNL.
● Single-mode fibre is used for long-distance The merger is expected to yield better synergies
transmission, while multimode fibre is and coordination.
used for shorter distances. The outer Additional info:
cladding of these fibres needs better
About BharatNet:
protection than metal wires.
• BharatNet Project was originally launched
in 2011 as the National Optical Fibre
93. Consider the following statements with Network(NOFN) and renamed as Bharat-
reference to BharatNet project: Net in 2015.
1. The aim of the project is to connect gram • It seeks to provide connectivity to 2.5 lakh
panchayats through broadband Gram Panchayats (GPs) through optical
2. It is a flagship mission implemented by fibre.
Bharat Broadband Network Ltd. (BBNL). • It is a flagship mission implemented by
3. BharatNet is the first ever project to achieve Bharat Broadband Network Ltd. (BBNL).
its target before the end of timeline. • The objective is to facilitate the delivery of
Which of the statements given above is/are e-governance, e-health, e-education, e-
correct? banking, Internet and other services to
rural India.
A. 1 only
The larger vision of the project is:
B. 2 and 3 only
C. 3 only
• To establish a highly scalable network 1. Quantum dots (QDs) are man-made
infrastructure accessible on a non- nanoscale crystals that can transport
discriminatory basis. electrons.
• To provide on demand, affordable 2. The properties of a quantum dot are only
broadband connectivity of 2 Mbps to 20 determined by its size
Mbps for all households and on demand Which of the statements given above is/are
capacity to all institutions. correct?
• To realise the vision of Digital India, in A. 1 only
partnership with States and the private
B. 2 only
sector.
C. Both 1 and 2

94. Which of the following is/are the D. Neither 1 nor 2


characteristics of Lasers? Answer: A
1. Superior Monochromatism Explanation:
2. Superior Directivity Statement 1 is correct: Quantum dots (QDs) are
3. Superior Coherence man-made nanoscale crystals that can transport
electrons.
4. High Output
Statement 2 is incorrect: The properties of a
Select the correct code: quantum dot are not only determined by its size
A. 1 and 2 only but also by its shape
B. 3 only Additional info:
C. 1 and 4 only What are quantum dots?
D. 1,2,3 and 4 ● Quantum dots (QDs) are man-made
nanoscale crystals that can transport
Answer: D
electrons.
Explanation:
● When UV light hits these semiconducting
We can separate the characteristics of laser beam nanoparticles, they can emit light of
into four major categories as: various colors.
• Superior Monochromatism ● These artificial semiconductor
• Superior Directivity nanoparticles that have found applications
in composites, solar cells and fluorescent
• Superior Coherence
biological labels.
• High Output
● Nanoparticles of semiconductors –
quantum dots – were theorized in the 1970s
95. Consider the following statements with and initially created in the early 1980s.
reference to Quantum Dots:
● If semiconductor particles are made small Additional information:
enough, quantum effects come into play, What is Biotechnology?
which limit the energies at which electrons
Biotechnology uses living cells to develop or
and holes (the absence of an electron) can
manipulate products for specific purposes, such as
exist in the particles.
genetically modified foods. Biotechnology is thus
● As energy is related to wavelength (or linked to genetic engineering and emerged as a
color), this means that the optical field in its own right at the beginning of the 20th
properties of the particle can be finely century in the food industry, which was later
tuned depending on its size. joined by other sectors such as medicine and the
● Thus, particles can be made to emit or environment.
absorb specific wavelengths (colors) of Types of Biotechnology
light, merely by controlling their size.
1. Red biotechnology.: This is the health
branch and responsible, according to the
96. Consider the following types of Biotechnology Innovation Organization
Biotechnologies: (BIO), for the development of more than
250 vaccines and medications such as
1. Red Biotechnology: Industrial
antibiotics, regenerative therapies and the
Biotechnology
production of artificial organs.
2. White Biotechnology: Medical
2. Green biotechnology. It is used by more
Biotechnology
than 13 million farmers worldwide to fight
3. Green Biotechnology: Agricultural pests and nourish crops and strengthen
Biotechnology them against microorganisms and extreme
4. Blue Biotechnology: Marine Biotechnology weather events, such as droughts and
frosts.
Which of the above is/are correctly matched?
3. White biotechnology. The industrial
A. 1 and 2 only
branch works to improve manufacturing
B. 3 and 4 only processes, the development of biofuels and
C. 1 and 3 only other technologies to make industry more
efficient and sustainable.
D. 2 and 4 only
4. Yellow biotechnology. This branch is
Answer: B
focused on food production and, for
Explanation: example, it carries out research to reduce
Pairs 1 and 2 are interchanged as shown below: the levels of saturated fats in cooking oils.
Its main function is to genetically improve
Red Biotechnology: Medical Biotechnology
products so that there is a higher quantity
White Biotechnology: Industrial Biotechnology or quality of food
Green Biotechnology: Agricultural Biotechnology 5. Blue biotechnology. This exploits marine
Blue Biotechnology: Marine Biotechnology resources to obtain aquaculture, cosmetics
and health care products. At the
environmental level, the aim is to preserve main activities are related to biological
marine species and ecosystems. In warfare and bioterrorism.
addition, it is the branch most widely used
to obtain biofuels from certain microalgae.
97. Which of the following is/are Gene editing
6. Grey biotechnology. Its purpose is the techniques?
conservation and restoration of
1. Zinc Finger Nucleases
contaminated natural ecosystems through,
as mentioned above, bioremediation 2. TALENs Gene Editing
processes. 3. CRISPR-Cas9 Gene Editing
7. Gold biotechnology. Also known as Select the correct code:
bioinformatics, it is responsible for
A. 1 only
obtaining, storing, analysing and
separating biological information, B. 2 and 3 only
especially that related to DNA and amino C. 3 only
acid sequences.
D. 1, 2 and 3
To these typologies, four further sub-categories
with corresponding colours have more recently Answer: D
been added: Explanation:
1. Brown biotechnology. This comes from Restriction Enzymes: The Original Genome Editor
green biotechnology with the aim of taking • The ability to edit genes became a
advantage of arid and desert soils to reality with the discovery of
include highly resistant plant species that restriction enzymes in the 1970s.
increase the flora and biodiversity of these
environments. • Restriction enzymes recognize
specific patterns of nucleotide
2. Purple biotechnology. It deals with the sequences and cut at that site,
legal study of the very aspects of this
science. They are closely related to • presenting an opportunity to insert
intellectual property, patents and the new DNA material at that location.
biosafety of processes involving living Zinc Finger Nucleases (ZFNs)
organisms. • Recognized the site better to edit, as
3. Orange biotechnology. This includes the off-target effects could be deleterious.
dissemination of information of interest to TALENs Gene Editing: Single Nucleotide
the other branches. it is carried out both in Resolution
the fields of education and scientific
CRISPR-Cas9 Gene Editing: Genome Editing
dissemination with new advances in
Revolutionized
biotechnologies.
• There are several gene editing
4. Black biotechnology. This includes all
methods, but a tool called CRISPR-
research work on micro-organisms that can
Cas9 has sparked a boom in research
be manipulated to attack human health. Its
as laboratories worldwide adopted it regulatory sequences, and non-coding
over the past five years because it’s RNAs reside.
faster, cheaper, simple to use with
minimal training and allows
manipulation of multiple genes at the Statement 2 is correct: Researchers investigating
same time. the “dark genome” report that they have
discovered recently evolved regions that code for
• Crisper-scan the genome looking for proteins associated with schizophrenia and bipolar
right location disorder. They say these new proteins can be used
• Then Cas9 protein(enzyme) as as biological indicators to distinguish between the
molecular scissor two conditions, and to identify patients more
prone to psychosis or suicide.

98. Consider the following statements: Additional information:

1. DNA present outside our genes in the • Hotspots in the dark genome associated
genomic space, which does not encode the with the disorders may have evolved
proteins is called the Dark Genome. because they have beneficial functions in
human development, but their disruption
2. Dark Genomes have evolved portions of by environmental factors leads to
coding proteins which can be used to susceptibility to, or development of,
distinguish between schizophrenia and schizophrenia or bipolar disorder.
bipolar disorders.
• The researchers think that these genomic
Which of the above statements is/are correct? components of schizophrenia and bipolar
A. 1 only disorder are specific to humans—the newly
B. 2 only discovered regions are not found in the
genomes of other vertebrates.
C. Both 1 and 2
D. Neither 1 nor 2
99. Consider the following statements:
Answer: C
1. In Artificial Intelligence Model based
Explanation: agent deals with the happy and
Statement 1 is correct: unhappy state
Dark Genome 2. An Artificial Intelligence system that is
• The human genome is conventionally created to complete a particular task is
divided into the “coding” genome, which known as Strong AI
generates the ~20,000 annotated human Which of the statements given above is/are
protein coding genes, and the “dark” correct?
genome, which does not encode proteins. A. 1 only
• The dark genome is a vast space, B. 2 only
accounting for the ~98.5% of genomic space
where repeat elements, enhancers, C. Both 1 and 2
D. Neither 1 nor 2 developed and taught to do a single job.
Answer: D Weak Al is used by industrial robots and
virtual personal assistants like Apple's Siri.
Explanation:
● Strong artificial intelligence (Al), often
Statement 1 is incorrect:
known as artificial general intelligence
● According to computer science, Artificial (AGI), refers to programming that can
Intelligence is the study of "intelligent mimic the cognitive capacities of the
agents" or "rational agents and their human brain.
environment".
● Agent: Agent is those that work or act in 100. Which of the following is/are the
their environment. Environment: It is the applications of Machine Learning?
surrounding of an agent. An agent takes
1. Sales forecasting for different products
input from the environment through
sensors & delivers the output to the 2. Fraud analysis in banking
environment through actuators. 3. Product recommendations
● Simple reflex agent acts on the basis of 4. Stock price prediction
current percept and ignores percept history
i.e. all the history an agent has perceived till Select the correct code:
date. Model-based agents have a model i.e. A. 1 and 2 only
knowledge about how things happen in the
B. 4 only
whole world & based on the model it
performs actions. C. 2 and 3 only

● Learning agents learn from past D. 1, 2, 3 and 4


experiences and automatically adapt Answer: D
through learning. It has learning
Explanation:
capabilities. Utility-based agents choose
actions on the basis of utilities or preference Applications of Machine learning:
and chooses the actions that maximize the ● Speech recognition: Machine learning is
expected utility. extensively used in developing speech
● Utility describes the happiness of the agent. recognition software applications like Siri
Hence, Utility-based agents deal with or Alexa. These speech recognition
happy & unhappy agents. applications enable machines to recognize,
understand, and interpret human speech,
Statement 2 is incorrect: making it easier for us to interface with
● An Artificial Intelligence system that is technology.
created to complete a particular task is
● Image processing: Machine learning has
known as Weak artificial intelligence.
great use in image processing applications.
● Weak Al, also known as narrow Al, is a It helps in object detection, object
type of artificial intelligence that is segmentation, and image recognition.
Applications of this technology are seen in ● Machine learning is a discipline of
security cameras that can recognize computer science that uses computer
people’s faces and cars that can detect algorithms and analytics to build
dangerous driving conditions. predictive models that can solve business
problems.
● Fraud detection: Machine learning
algorithms can be used to detect anomalies
and fraud in various financial transactions.
For instance, credit card companies use
machine learning to detect unusual
patterns of usage that could indicate fraud.
● Medical diagnosis: Doctors can use
machine learning algorithms to analyze
large amounts of medical data and
interpret the results. This information can
be extremely useful in diagnosing diseases
or predicting diseases that may occur in the
future.
● Recommendation systems:
Recommendation systems are becoming
increasingly popular in e-commerce and
entertainment applications. Machine
learning algorithms can be used to analyze
user data and provide recommendations
for products or services based on their
previous purchases and interests.
● Customer service: Machine learning can
help automate customer service
applications by providing users with
personalized recommendations or
responses based on their previous
interactions.
● Natural language processing: Natural
language processing is a field of machine
learning used to build intelligent chatbots
and virtual assistants. These applications
can answer questions, make
recommendations, and provide
personalized assistance to users.

You might also like